Журнал вычислительной математики и математической физики, 2019, T. 59, № 9, стр. 1495-1515

Автоволновые процессы в диффузионных нейронных системах

С. Д. Глызин 1*, А. Ю. Колесов 1**, Н. Х. Розов 2***

1 ЯрГУ, матем. ф-т
150000 Ярославль, ул. Советская, 14, Россия

2 МГУ, механ.-матем. ф-т
119899 Москва, Ленинские горы, Россия

* E-mail: glyzin@uniyar.ac.ru
** E-mail: andkolesov@mail.ru
*** E-mail: fpo.mgu@mail.ru

Поступила в редакцию 25.04.2019
После доработки 25.04.2019
Принята к публикации 15.05.2019

Полный текст (PDF)

Аннотация

Рассматривается так называемая диффузионная нейронная модель, представляющая собой систему из $m,$ $m \geqslant 2$ одинаковых нелинейных дифференциальных уравнений с запаздыванием, связанных посредством линейных диффузионных слагаемых. Устанавливается, что при подходящем выборе коэффициента диффузии в упомянутой системе существует набор из $m$ устойчивых релаксационных циклов. Библ. 7. Фиг. 3.

Ключевые слова: билокальная модель, автоволновые процессы, асимптотика, устойчивость, диффузионная система.

1. БИЛОКАЛЬНАЯ МОДЕЛЬ

Согласно работам [1], [2] функционирование отдельного нейрона моделируется уравнением вида

(1.1)
$\dot {u} = \lambda f(u(t - 1))u$
для мембранного потенциала $u = u(t) > 0.$ Здесь параметр $\lambda > 0,$ характеризующий скорость протекания электрических процессов в нейроне, предполагается большим, точка – дифференцирование по $t,$ а функция $f(u) \in {{C}^{2}}({{\mathbb{R}}_{ + }})$, ${{\mathbb{R}}_{ + }} = \left\{ {u \in \mathbb{R}:u \geqslant 0} \right\}$, обладает свойствами
(1.2)
$f(0) = 1,\quad f(u) = - a + O\left( {\frac{1}{u}} \right),\quad uf{\kern 1pt} {\text{'}}(u) = O\left( {\frac{1}{u}} \right),\quad {{u}^{2}}f{\kern 1pt} {\text{''}}(u) = O\left( {\frac{1}{u}} \right)$
при $u \to + \infty ,$ где $a = {\text{const}} > 0.$ Примером такой функции служит

(1.3)
$f(u) = \frac{{1 - u}}{{1 + u{\text{/}}a}}.$

Уравнение (1.1), представляющее собой некоторую модификацию известного уравнения Хатчинсона [3], было предложено и исследовано в статье [4]. В упомянутой работе показано, что при $\lambda \gg 1$ оно допускает экспоненциально орбитально устойчивый цикл ${{u}_{0}}(t,\lambda ) > 0$, ${{u}_{0}}(0,\lambda ) \equiv 1$, периода ${{T}_{0}}(\lambda ),$ удовлетворяющий предельным соотношениям:

(1.4)
$\mathop {lim}\limits_{\lambda \to + \infty } {{T}_{0}}(\lambda ) = {{T}_{0}},\quad \mathop {max}\limits_{0 \leqslant t \leqslant {{T}_{0}}(\lambda )} \left| {{{\omega }_{0}}(t,\lambda ) - {{\omega }_{0}}(t)} \right| = O\left( {\frac{1}{\lambda }} \right),\quad \lambda \to + \infty ,$
где ${{T}_{0}} = (1 + a){{t}_{0}}$, ${{t}_{0}} = 1 + 1{\text{/}}a$, ${{\omega }_{0}}(t,\lambda ) = (1{\text{/}}\lambda )ln{{u}_{0}}(t,\lambda )$, а ${{T}_{0}}$-периодическая функция ${{\omega }_{0}}(t)$ задается равенствами
(1.5)
${{\omega }_{0}}(t) = \left\{ \begin{gathered} t\quad {\text{при}}\quad 0 \leqslant t \leqslant 1, \hfill \\ 1 - a(t - 1)\quad {\text{при}}\quad 1 \leqslant t \leqslant {{t}_{0}} + 1, \hfill \\ t - {{T}_{0}}\quad {\text{при}}\quad {{t}_{0}} + 1 \leqslant t \leqslant {{T}_{0}}. \hfill \\ \end{gathered} \right.\quad {{\omega }_{0}}(t + {{T}_{0}}) \equiv {{\omega }_{0}}(t),$
Наглядное представление о релаксационных свойствах этого цикла дает его график на плоскости $(t,u)$, построенный численно для случая (1.1), (1.3) при $\lambda = 5$, $a = 2$ (см. фиг. 1).

Фиг. 1.

Предположим теперь, что имеется система из двух электрически связанных нейронов. В этом случае их мембранные потенциалы ${{u}_{1}}(t)$, ${{u}_{2}}(t)$ удовлетворяют системе дифференциально-разностных уравнений вида

(1.6)
$\begin{gathered} \mathop {\dot {u}}\nolimits_1 = D({{u}_{2}} - {{u}_{1}}) + \lambda f({{u}_{1}}(t - 1)){{u}_{1}}, \\ \mathop {\dot {u}}\nolimits_2 = D({{u}_{1}} - {{u}_{2}}) + \lambda f({{u}_{2}}(t - 1)){{u}_{2}}, \\ \end{gathered} $
где параметр $D > 0,$ который уместно назвать коэффициентом диффузии, характеризует глубину связи между нейронами. Саму же систему (1.6) принято называть билокальной моделью.

Одним из стационарных режимов системы (1.6) является однородный цикл

(1.7)
$({{u}_{1}},{{u}_{2}}) = ({{u}_{0}}(t,\lambda ),{{u}_{0}}(t,\lambda )),$
где ${{u}_{0}}(t,\lambda )$ – периодическое решение уравнения (1.1) с асимптотикой (1.4), (1.5). Другие же стационарные режимы этой системы, отличные от цикла (1.7), по аналогии с пространственно непрерывным случаем [5] будем называть автоволновыми процессами или автоволнами.

В настоящей работе продолжается начатое в [6], [7] исследование автоволновых режимов системы (1.6). А именно, ниже при условиях

(1.8)
$D = \lambda exp( - b\lambda ),\quad b = {\text{const}} > 0,\quad \lambda \gg 1,$
(1.9)
$a > 1,\quad 0 < b < 1$
устанавливается существование у билокальной модели (1.6) двух устойчивых автоволновых периодических решений
(1.10)
$({{u}_{1}},{{u}_{2}}) = ({{u}_{{1, * }}}(t,\lambda ),{{u}_{{2, * }}}(t,\lambda )),\quad ({{u}_{1}},{{u}_{2}}) = ({{u}_{{2, * }}}(t,\lambda ),{{u}_{{1, * }}}(t,\lambda )),\quad {{u}_{{1, * }}}(0,\lambda ) \equiv 1,$
переходящих друг в друга при замене переменных $({{u}_{1}},{{u}_{2}}) \to ({{u}_{2}},{{u}_{1}})$. Изучаются также асимптотические свойства этих циклов.

Введем в рассмотрение момент времени $t = {{t}_{ * }}$, ${{t}_{ * }} = b{\text{/}}a$, принадлежащий в силу (1.9) интервалу $(0,{{t}_{0}} - 1)$. Как оказывается, ${{T}_{0}}$-периодические функции ${{\omega }_{0}}(t)$, ${{\omega }_{0}}(t - {{t}_{ * }})$ (см. (1.5)) служат нулевыми приближениями при $\lambda \to + \infty $ для функций

(1.11)
${{\omega }_{{j, * }}}(t,\lambda ) = \frac{1}{\lambda }ln{{u}_{{j, * }}}(t,\lambda ),\quad j = 1,2,$
где ${{u}_{{j, * }}}(t,\lambda )$, $j = 1,2$ – компоненты некоторых циклов вида (1.10). Точнее говоря, справедливы следующие утверждения.

Теорема 1.1. Найдется такое достаточно большое ${{\lambda }_{0}} > 0$, что при всех $\lambda \geqslant {{\lambda }_{0}}$ и при условиях (1.8), (1.9) система (1.6) допускает пару автоволновых циклов (1.10). При $\lambda \to + \infty $ для периода ${{T}_{ * }}(\lambda )$ этих циклов и для соответствующих им функций (1.11) выполняются асимптотические представления

(1.12)
$\begin{gathered} {{T}_{ * }}(\lambda ) = {{T}_{0}} + O\left( {\frac{{ln\lambda }}{{{{\lambda }^{2}}}}} \right),\quad \mathop {max}\limits_{0 \leqslant t \leqslant {{T}_{ * }}(\lambda )} \left| {{{\omega }_{{1, * }}}(t,\lambda ) - {{\omega }_{0}}(t)} \right| = O\left( {\frac{1}{\lambda }} \right), \\ \mathop {max}\limits_{0 \leqslant t \leqslant {{T}_{ * }}(\lambda )} \left| {{{\omega }_{{2, * }}}(t,\lambda ) - {{\omega }_{0}}(t - {{t}_{ * }})} \right| = O\left( {\frac{{ln\lambda }}{\lambda }} \right). \\ \end{gathered} $

Теорема 1.2. Циклы (1.10), о которых идет речь в предыдущей теореме, экспоненциально орбитально устойчивы.

Доказательство теоремы 1.1 связано с асимптотическим анализом оператора последования Пуанкаре по траекториям системы (1.6). Как будет показано в дальнейшем, этот оператор преобразует в себя некоторое замкнутое, ограниченное, выпуклое множество начальных функций и является компактным. Тем самым, согласно принципу Шаудера, он допускает неподвижную точку.

При обосновании теоремы 1.2 следует иметь в виду, что в силу инвариантности системы (1.6) по отношению к замене переменных $({{u}_{1}},{{u}_{2}}) \to ({{u}_{2}},{{u}_{1}})$ свойства устойчивости пары циклов (1.10) одинаковы. Поэтому достаточно провести асимптотический анализ системы в вариациях на первом из этих циклов и убедиться в его устойчивости.

Согласно формулам (1.5), (1.12) эволюция во времени компонент ${{u}_{1}}$, ${{u}_{2}}$ циклов (1.10) характеризуется чередованием асимптотически высоких (порядка $O(exp\lambda )$) всплесков длительности $\Delta t \sim 1 + 1{\text{/}}a$ с промежутками времени, на которых ${{u}_{1}},$ ${{u}_{2}}$ асимптотически малы (имеют порядок $O(exp( - q\lambda ))$, $q = {\text{const}} > 0$). Кроме того, всплески ${{u}_{1}}$ и ${{u}_{2}}$ сдвинуты во времени по отношению друг к другу на величину ${{t}_{ * }}$.

Перечисленные факты иллюстрирует фиг. 2, где на плоскости $(t,u)$ изображены графики компонент ${{u}_{1}}(t)$, ${{u}_{2}}(t)$ одного из этих режимов в случае (1.3), (1.8) при $\lambda = 8$, $a = 2$, $b = 0.9$ (сплошной линией показан график ${{u}_{1}}(t)$, а штриховой – график ${{u}_{2}}(t)$).

Фиг. 2.

2. ДОКАЗАТЕЛЬСТВО ТЕОРЕМЫ 1.1

2.1. Общая схема исследования

При условиях (1.8), (1.9) сделаем в исходной системе (1.6) замены ${{u}_{j}} = exp(\lambda {{\omega }_{j}})$, $j = 1,2$, и положим $\varepsilon = 1{\text{/}}\lambda \ll 1$. В результате для новых переменных ${{\omega }_{j}} = {{\omega }_{j}}(t)$, $j = 1,2$, приходим к системе

(2.1)
$\begin{gathered} \mathop {\dot {\omega }}\nolimits_1 = exp\left( {\frac{{{{\omega }_{2}} - {{\omega }_{1}} - b}}{\varepsilon }} \right) - exp\left( { - \frac{b}{\varepsilon }} \right) + f\left( {exp\left( {\frac{{{{\omega }_{1}}(t - 1)}}{\varepsilon }} \right)} \right), \\ \mathop {\dot {\omega }}\nolimits_2 = exp\left( {\frac{{{{\omega }_{1}} - {{\omega }_{2}} - b}}{\varepsilon }} \right) - exp\left( { - \frac{b}{\varepsilon }} \right) + f\left( {exp\left( {\frac{{{{\omega }_{2}}(t - 1)}}{\varepsilon }} \right)} \right), \\ \end{gathered} $
более удобной для дальнейшего анализа.

Перейдем к описанию класса начальных условий для системы (2.1). В связи с этим фиксируем достаточно малую постоянную ${{\sigma }_{0}} > 0$ (в последующем на нее будет наложено некоторое ограничение) и рассмотрим банахово пространство $E = C\left( {\left[ { - 1 - {{\sigma }_{0}}, - {{\sigma }_{0}}} \right];{{\mathbb{R}}^{2}}} \right)$ непрерывных по $t \in [ - 1 - {{\sigma }_{0}}, - {{\sigma }_{0}}]$ вектор-функций $\varphi (t) = {\text{colon}}({{\varphi }_{1}}(t),{{\varphi }_{2}}(t))$ со значениями в ${{\mathbb{R}}^{2}}$. Норму в $E$ зададим обычным образом, т.е. посредством формулы

(2.2)
$\left\| \varphi \right\| = \mathop {max}\limits_{j = 1,2} \mathop {max}\limits_{ - 1 - {{\sigma }_{0}} \leqslant t \leqslant - {{\sigma }_{0}}} \left| {{{\varphi }_{j}}(t)} \right|.$
Далее, интересующее нас множество начальных функций $S \subset E$ определим равенством
(2.3)
$S = \left\{ {\varphi (t) = \operatorname{colon} ({{\varphi }_{1}}(t),{{\varphi }_{2}}(t)):} \right. - {\kern 1pt} {{q}_{1}} \leqslant {{\varphi }_{1}}(t) \leqslant - {{q}_{2}},\quad - {\kern 1pt} {{q}_{3}} \leqslant {{\varphi }_{2}}(t) \leqslant - {{q}_{4}},\;{{\varphi }_{1}}( - {{\sigma }_{0}}) = - {{\sigma }_{0}},\quad \left. {{{z}_{\varphi }} \in \Omega } \right\}.$
Здесь ${{q}_{1}} > {{q}_{2}} > 0$, ${{q}_{3}} > {{q}_{4}} > 0$ – некоторые универсальные (не зависящие от $t$, $\varphi $, $\varepsilon $) постоянные, выбором которых распорядимся в дальнейшем, через ${{z}_{\varphi }}$ обозначен функционал
(2.4)
${{z}_{\varphi }} = \frac{1}{\varepsilon }\left( {{{\varphi }_{2}}( - {{\sigma }_{0}}) + {{\sigma }_{0}} + \frac{b}{a} - \frac{\varepsilon }{a}ln\frac{1}{\varepsilon }} \right),$
а множество $\Omega $ представляет собой отрезок вида

(2.5)
$\Omega \;\mathop = \limits^{{\text{def}}} \;\left[ {{{z}_{1}},{{z}_{2}}} \right],\quad {{z}_{1}},{{z}_{2}} = {\text{const}} \in \mathbb{R},\quad {{z}_{1}} < {{z}_{2}}.$

Рассмотрим решение ${{\omega }_{\varphi }}(t,\varepsilon ) = \operatorname{colon} ({{\omega }_{{1,\varphi }}}(t,\varepsilon ),{{\omega }_{{2,\varphi }}}(t,\varepsilon ))$, $t \geqslant - {{\sigma }_{0}}$, системы (2.1) с произвольным начальным условием $\varphi (t) \in S$ при $t \in [ - 1 - {{\sigma }_{0}}, - {{\sigma }_{0}}]$, обозначим через $t = {{T}_{\varphi }}(\varepsilon )$ второй положительный корень уравнения

(2.6)
${{\omega }_{{1,\varphi }}}(t - {{\sigma }_{0}},\varepsilon ) = - {{\sigma }_{0}}$
(если он существует) и зададим оператор $\Pi ,$ действующий из $S$ в $E$ по правилу
(2.7)
$\Pi (\varphi ) = {{\omega }_{\varphi }}(t + {{T}_{\varphi }}(\varepsilon ),\varepsilon ),\quad - {\kern 1pt} 1 - {{\sigma }_{0}} \leqslant t \leqslant - {{\sigma }_{0}}.$
Как будет показано в последующем, при подходящем выборе параметров ${{\sigma }_{0}}$, ${{q}_{j}}$, $j = 1,\; \ldots ,\;4$, и ${{z}_{1}}$, ${{z}_{2}}$ оператор (2.7) определен на множестве (2.3) и, более того, $\Pi (S) \subset S$, ${{T}_{\varphi }}(\varepsilon ) > 1$ при всех $\varphi \in S$. Далее, поскольку множество $S$ замкнуто, ограничено и выпукло, а оператор $\Pi $ в силу неравенства ${{T}_{\varphi }} > 1$ компактен, то согласно принципу Шаудера он имеет в $S$ хотя бы одну неподвижную точку $\varphi = {{\varphi }_{ * }}(t,\varepsilon )$. Ясно также, что решение $\omega (t,\varepsilon ) = {{\left. {{{\omega }_{\varphi }}} \right|}_{{\varphi = {{\varphi }_{ * }}}}}$ системы (2.1) является периодическим с периодом $T(\varepsilon ) = {{\left. {{{T}_{\varphi }}} \right|}_{{\varphi = {{\varphi }_{ * }}}}}$. Остается заметить, что это решение порождает в исходной системе (1.6), (1.8) требуемую пару циклов (1.10).

2.2. Асимптотический анализ оператора последования Пуанкаре

Обратимся теперь непосредственно к доказательству теоремы 1.1. Забегая вперед, отметим, что, во-первых, для реализации описанной в предыдущем пункте программы действий достаточно знать равномерную по $\varphi \in S$ асимптотику при $\varepsilon \to 0$ решения ${{\omega }_{\varphi }}(t,\varepsilon )$ на промежутке времени

(2.8)
$ - {{\sigma }_{0}} \leqslant t \leqslant {{T}_{0}} - {{\sigma }_{0}}{\text{/}}2;$
во-вторых, на отрезке (2.8) имеет место равномерная по $t$, $\varphi $ асимптотическая близость компонент ${{\omega }_{{1,\varphi }}}(t,\varepsilon )$, ${{\omega }_{{2,\varphi }}}(t,\varepsilon )$ к функциям ${{\omega }_{0}}(t)$ и ${{\omega }_{0}}(t - {{t}_{ * }})$ соответственно. При обосновании данной близости в отдельном рассмотрении нуждаются окрестности особых точек $t = 1$, $t = {{t}_{ * }} + 1$, $t = {{t}_{0}} + 1$, $t = {{t}_{{ * * }}} + 1$, где ${{t}_{{ * * }}} = {{t}_{0}} + {{t}_{ * }}$, отвечающих изломам графиков ${{\omega }_{0}}(t)$ и ${{\omega }_{0}}(t - {{t}_{ * }})$.

Для удобства восприятия приведенных ниже асимптотических разложений уместно сначала обратиться к фиг. 3, где представлено взаимное расположение графиков функций ${{\omega }_{0}}(t)$, ${{\omega }_{0}}(t - {{t}_{ * }})$ и отмечены ключевые моменты времени ${{t}_{ * }} < 1 < {{t}_{ * }} + 1 < {{t}_{0}} < {{t}_{{ * * }}} < {{t}_{0}} + 1 < {{t}_{{ * * }}} + 1$.

Фиг. 3.

Оговорим сразу выбор фигурирующего в (2.3) параметра ${{\sigma }_{0}}$. По чисто техническим причинам удобно считать, что

(2.9)
${{\sigma }_{0}} < \frac{1}{2}min({{t}_{ * }},1 - {{t}_{ * }}).$
Кроме того, предположим, что на отрезке (2.8) выполняется априорное условие
(2.10)
${{\omega }_{{1,\varphi }}}(t,\varepsilon ) - {{\omega }_{{2,\varphi }}}(t,\varepsilon ) - b \leqslant - M$
с некоторой не зависящей от $t$, $\varphi $, $\varepsilon $ постоянной $M > 0$.

Принимая во внимание неравенство (2.10), убеждаемся в том, что компонента ${{\omega }_{{2,\varphi }}}(t,\varepsilon )$ интересующего нас решения ${{\omega }_{\varphi }}(t,\varepsilon )$ системы (2.1) удовлетворяет уравнению вида

(2.11)
$\mathop {\dot {\omega }}\nolimits_2 = f\left( {exp\left( {\frac{{{{\omega }_{2}}(t - 1)}}{\varepsilon }} \right)} \right) + O\left( {exp\left( { - \frac{q}{\varepsilon }} \right)} \right)$
(здесь и в последующем одной и той же буквой $q$ обозначаются различные не зависящие от $t$, $\varphi $, $\varepsilon $ положительные константы, точные значения которых несущественны). Напомним, далее, что уравнение, получающееся из (2.11) после отбрасывания добавки порядка $O(exp( - q{\text{/}}\varepsilon ))$, асимптотически проинтегрировано в статье [4]. Опираясь на результаты этой статьи, для ${{\omega }_{{2,\varphi }}}(t,\varepsilon )$ приходим к следующей серии асимптотических формул.

1. На отрезке $ - {{\sigma }_{0}} \leqslant t \leqslant {{t}_{ * }}(\varepsilon ,z) + 1 - {{\sigma }_{0}}$, где

(2.12)
${{t}_{ * }}(\varepsilon ,z) = {{t}_{ * }} - \frac{\varepsilon }{a}ln\frac{1}{\varepsilon } - \varepsilon z,$
при $\varepsilon \to 0$ выполняется равномерное по $t$, $\varphi $ асимптотическое равенство
(2.13)
${{\omega }_{{2,\varphi }}}(t,\varepsilon ) = t - {{t}_{ * }}(\varepsilon ,z) + O\left( {exp\left( { - \frac{q}{\varepsilon }} \right)} \right).$
Что же касается величины $z$, то она представляет собой значение функционала (2.4) на соответствующей начальной функции $\varphi \in S$. Однако, как в случае (2.12), так и всюду ниже, индекс $\varphi $ у ${{z}_{\varphi }}$ для краткости мы опускаем.

2. На отрезке ${{t}_{ * }}(\varepsilon ,z) + 1 - {{\sigma }_{0}} \leqslant t \leqslant {{t}_{ * }}(\varepsilon ,z) + 1 + {{\sigma }_{0}}$ при $\varepsilon \to 0$ справедливо равномерное по $t$, $\varphi $ асимптотическое равенство

(2.14)
${{\omega }_{{2,\varphi }}}(t,\varepsilon ) = {{\left. {1 + \varepsilon {{w}_{0}}(\tau )} \right|}_{{\tau = (t - {{t}_{ * }}(\varepsilon ,z) - 1)/\varepsilon }}} + O\left( {exp\left( { - \frac{q}{\varepsilon }} \right)} \right),$
где
(2.15)
${{w}_{0}}(\tau ) = \tau + \int\limits_{ - \infty }^\tau {\left[ {f(exps) - 1} \right]} {\kern 1pt} ds.$
Отметим еще, что
(2.16)
${{w}_{0}}(\tau ) = \tau + O(exp\tau ),\quad \tau \to - \infty ;\quad {{w}_{0}}(\tau ) = - a\tau + {{c}_{0}} + O(exp( - \tau )),\quad \tau \to + \infty ,$
где

(2.17)
${{c}_{0}} = \int\limits_0^1 {\frac{{f(u) - 1}}{u}} du + \int\limits_1^{ + \infty } {\frac{{f(u) + a}}{u}} du.$

3. На промежутке времени ${{t}_{ * }}(\varepsilon ,z) + 1 + {{\sigma }_{0}} \leqslant t \leqslant {{t}_{{ * * }}}(\varepsilon ,z) + 1 - {{\sigma }_{0}}$, где ${{t}_{{ * * }}}(\varepsilon ,z) = {{t}_{0}} + {{t}_{ * }}(\varepsilon ,z)$, при $\varepsilon \to 0$ имеет место равномерное по $t$, $\varphi $ асимптотическое представление

(2.18)
${{\omega }_{{2,\varphi }}}(t,\varepsilon ) = 1 - a(t - {{t}_{ * }}(\varepsilon ,z) - 1) + \varepsilon {{c}_{0}} + O\left( {exp\left( { - \frac{q}{\varepsilon }} \right)} \right).$
Здесь ${{c}_{0}}$ – постоянная (2.17).

4. На отрезке ${{t}_{{ * * }}}(\varepsilon ,z) + 1 - {{\sigma }_{0}} \leqslant t \leqslant {{t}_{{ * * }}}(\varepsilon ,z) + 1 + {{\sigma }_{0}}$ при $\varepsilon \to 0$ функция ${{\omega }_{{2,\varphi }}}(t,\varepsilon )$ допускает равномерное по $t$, $\varphi $ асимптотическое равенство

(2.19)
${{\omega }_{{2,\varphi }}}(t,\varepsilon ) = - a + {{\left. {\varepsilon {{w}_{1}}(\tau )} \right|}_{{\tau = (t - {{t}_{{ * * }}}(\varepsilon ,z) - 1)/\varepsilon }}} + O\left( {exp\left( { - \frac{q}{\varepsilon }} \right)} \right),$
где
(2.20)
${{w}_{1}}(\tau ) = - a\tau + {{c}_{0}} + \int\limits_{ - \infty }^\tau {\left[ {f(exp( - as + {{c}_{0}})) + a} \right]} ds.$
Что же касается функции (2.20), то для нее справедливы асимптотические формулы:

(2.21)
${{w}_{1}}(\tau ) = - a\tau + {{c}_{0}} + O(exp(a\tau )),\quad \tau \to - \infty ;\quad {{w}_{1}}(\tau ) = \tau + O(exp( - a\tau )),\quad \tau \to + \infty .$

5. На отрезке ${{t}_{{ * * }}}(\varepsilon ,z) + 1 + {{\sigma }_{0}} \leqslant t \leqslant {{T}_{0}} - {{\sigma }_{0}}{\text{/}}2$ при $\varepsilon \to 0$ равномерно по $t$, $\varphi $ имеем

(2.22)
${{\omega }_{{2,\varphi }}}(t,\varepsilon ) = t - {{T}_{0}} - {{t}_{ * }}(\varepsilon ,z) + O\left( {exp\left( { - \frac{q}{\varepsilon }} \right)} \right).$

Характерная особенность системы (2.1) состоит в том, что ее компоненты ${{\omega }_{1}},$ ${{\omega }_{2}}$ взаимодействуют между собой по принципу ”ведущий–ведомый”. Действительно, как уже было сказано выше, уравнение для ${{\omega }_{2}}$ с экспоненциальной точностью отщепляется от (2.1). Это обстоятельство позволило нам воспользоваться результатами из [4] и получить для ${{\omega }_{{2,\varphi }}}(t,\varepsilon )$ формулы (2.13)(2.22). Теперь же обратимся к первому уравнению из (2.1), считая в нем компоненту ${{\omega }_{2}} = {{\omega }_{{2,\varphi }}}(t,\varepsilon )$ уже известной функцией. Интегрируя данное уравнение методом шагов, т.е. рассматривая последовательно промежутки времени длины не более единицы, получаем очередную серию асимптотических формул. А именно, для компоненты ${{\omega }_{1}} = {{\omega }_{{1,\varphi }}}(t,\varepsilon )$ на этом пути выходят следующие равенства.

1. На отрезке времени $ - {{\sigma }_{0}} \leqslant t \leqslant 1 - {{\sigma }_{0}}$ при $\varepsilon \to 0$ справедливо равномерное по $t$, $\varphi $ асимптотическое представление

(2.23)
${{\omega }_{{1,\varphi }}}(t,\varepsilon ) = t + O\left( {exp\left( { - \frac{q}{\varepsilon }} \right)} \right).$

2. На промежутке $1 - {{\sigma }_{0}} \leqslant t \leqslant 1 + {{\sigma }_{0}}$ при $\varepsilon \to 0$ равномерно по $t$, $\varphi $ имеем

(2.24)
${{\omega }_{{1,\varphi }}}(t,\varepsilon ) = 1 + {{\left. {\varepsilon {{{v}}_{0}}(\tau )} \right|}_{{\tau = (t - 1)/\varepsilon }}} + O\left( {exp\left( { - \frac{q}{\varepsilon }} \right)} \right),$
где ${{{v}}_{0}}(\tau ) = {{w}_{0}}(\tau )$ (см. (2.15)).

3. На отрезке $1 + {{\sigma }_{0}} \leqslant t \leqslant {{t}_{ * }}(\varepsilon ,z) + 1 - {{\sigma }_{0}}$ (длина которого в силу (2.9) положительна) при $\varepsilon \to 0$ выполняется равномерное по $t$, $\varphi $ асимптотическое равенство

(2.25)
${{\omega }_{{1,\varphi }}}(t,\varepsilon ) = 1 - a(t - 1) + \varepsilon {{c}_{0}} + O\left( {exp\left( { - \frac{q}{\varepsilon }} \right)} \right),$
где ${{c}_{0}}$ – постоянная (2.17).

4. На отрезке ${{t}_{ * }}(\varepsilon ,z) + 1 - {{\sigma }_{0}} \leqslant t \leqslant {{t}_{ * }}(\varepsilon ,z) + 1 + {{\sigma }_{0}}$ компонента ${{\omega }_{{1,\varphi }}}(t,\varepsilon )$ допускает при $\varepsilon \to 0$ равномерное по $t$, $\varphi $ асимптотическое представление

(2.26)
${{\omega }_{{1,\varphi }}}(t,\varepsilon ) = 1 - b + {{\left. {\varepsilon {{{v}}_{1}}(\tau ,\varepsilon )} \right|}_{{\tau = (t - {{t}_{ * }}(\varepsilon ,z) - 1)/\varepsilon }}} + O\left( {exp\left( { - \frac{q}{\varepsilon }} \right)} \right),$
где

(2.27)
${{{v}}_{1}}(\tau ,\varepsilon ) = - a\tau + ln\left( {\frac{{exp(az + {{c}_{0}})}}{\varepsilon } + \int\limits_{ - \infty }^\tau {exp\left[ {{{{v}}_{0}}(s) + as} \right]} {\kern 1pt} {\kern 1pt} ds} \right).$

5. На промежутке ${{t}_{ * }}(\varepsilon ,z) + 1 + {{\sigma }_{0}} \leqslant t \leqslant {{t}_{0}}(\varepsilon ,z) + 1 - {{\sigma }_{0}}$, где

(2.28)
${{t}_{0}}(\varepsilon ,z) = {{t}_{0}} + \frac{\varepsilon }{a}( - az + {{c}_{0}} + ln(exp(az) + {{t}_{0}} - {{t}_{ * }} - 1)),$
при $\varepsilon \to 0$ справедлива равномерная по $t$, $\varphi $ асимптотическая формула
(2.29)
$\begin{gathered} {{\omega }_{{1,\varphi }}}(t,\varepsilon ) = 1 - b - a(t - {{t}_{ * }}(\varepsilon ,z) - 1) + \varepsilon {{c}_{0}} + \\ + \;\varepsilon ln\left( {\frac{{exp(az) + t - {{t}_{ * }}(\varepsilon ,z) - 1}}{\varepsilon } + {{c}_{2}}} \right) + O\left( {exp\left( { - \frac{q}{\varepsilon }} \right)} \right). \\ \end{gathered} $
Здесь ${{c}_{2}} = {{c}_{1}}exp( - {{c}_{0}})$, а постоянная ${{c}_{1}}$ задается соотношением

(2.30)
${{c}_{1}} = \int\limits_{ - \infty }^0 {exp\left[ {{{{v}}_{0}}(s) + as} \right]} ds + \int\limits_0^{ + \infty } {(exp\left[ {{{{v}}_{0}}(s) + as} \right] - exp{{c}_{0}})} ds.$

6. На отрезке ${{t}_{0}}(\varepsilon ,z) + 1 - {{\sigma }_{0}} \leqslant t \leqslant {{t}_{0}}(\varepsilon ,z) + 1 + {{\sigma }_{0}}$ при $\varepsilon \to 0$ для компоненты ${{\omega }_{{1,\varphi }}}(t,\varepsilon )$ имеет место равномерное по $t$, $\varphi $ асимптотическое равенство

(2.31)
${{\omega }_{{1,\varphi }}}(t,\varepsilon ) = - a + {{\left. {\varepsilon {{{v}}_{2}}(\tau ,\varepsilon )} \right|}_{{\tau = (t - {{t}_{0}}(\varepsilon ,z) - 1)/\varepsilon }}} + O\left( {{{\varepsilon }^{2}}ln\frac{1}{\varepsilon }} \right),$
где

(2.32)
$\begin{gathered} {{{v}}_{2}}(\tau ,\varepsilon ) = - a\tau - ln(exp(az) + {{t}_{0}} - {{t}_{ * }} - 1) + lnK(\tau ) + \\ + \;ln\left( {exp(az) + {{t}_{0}} - {{t}_{ * }} + \varepsilon \tau + \varepsilon \int\limits_{ - \infty }^\tau {\left( {\frac{1}{{K(s)}} - 1} \right)} ds} \right), \\ \end{gathered} $
(2.33)
$K(\tau ) = exp\left\{ {\int\limits_{ - \infty }^\tau {\left[ {f(exp( - as)) + a} \right]} ds} \right\}.$

7. На отрезке ${{t}_{0}}(\varepsilon ,z) + 1 + {{\sigma }_{0}} \leqslant t \leqslant {{T}_{0}} - {{\sigma }_{0}}{\text{/}}2$ при $\varepsilon \to 0$ компонента ${{\omega }_{{1,\varphi }}}(t,\varepsilon )$ обладает равномерной по $t$, $\varphi $ асимптотикой

(2.34)
${{\omega }_{{1,\varphi }}}(t,\varepsilon ) = t - {{T}_{0}} + \varepsilon \left[ {z + ln(exp(az) + {{t}_{0}} - {{t}_{ * }}) - \left( {1 + \frac{1}{a}} \right)ln(exp(az) + {{t}_{0}} - {{t}_{ * }} - 1)} \right] + O\left( {{{\varepsilon }^{2}}ln\frac{1}{\varepsilon }} \right).$

Просуммируем полученные результаты. Объединяя формулы (2.13)(2.34), приходим к выводу, что при $\varepsilon \to 0$ справедливы равномерные по $t$ из отрезка (2.8) и по $\varphi \in S$ асимптотические представления

(2.35)
${{\omega }_{{1,\varphi }}}(t,\varepsilon ) = {{\omega }_{0}}(t) + O(\varepsilon ),\quad {{\omega }_{{2,\varphi }}}(t,\varepsilon ) = {{\omega }_{0}}(t - {{t}_{ * }}) + O\left( {\varepsilon ln\frac{1}{\varepsilon }} \right).$

Следует, однако, напомнить, что все наши построения носят условный характер, поскольку были проделаны в предположении (2.10). Проверим теперь справедливость и самого неравенства (2.10). С этой целью подставим формулы (2.35) (пока априорные) в (2.10) и учтем, что ${{\omega }_{0}}(t) - {{\omega }_{0}}(t - {{t}_{ * }}) - b < 0$ $\forall t \in [ - {{\sigma }_{0}},{{T}_{0}} - {{\sigma }_{0}}{\text{/}}2]$. В результате убеждаемся в том, что при всех достаточно малых $\varepsilon > 0$ требование (2.10) действительно выполняется с любой фиксированной постоянной

$M \in \left( {0,\mathop { - max}\limits_{ - {{\sigma }_{0}} \leqslant t \leqslant {{T}_{0}} - {{\sigma }_{0}}/2} \left[ {{{\omega }_{0}}(t) - {{\omega }_{0}}(t - {{t}_{ * }}) - b} \right]} \right).$
Таким образом, при указанном выборе $M$ все приведенные выше асимптотические представления для ${{\omega }_{{1,\varphi }}}(t,\varepsilon )$, ${{\omega }_{{2,\varphi }}}(t,\varepsilon )$ обретают законную силу.

Равенства (2.35) позволяют локализовать интересующий нас второй положительный корень $t = {{T}_{\varphi }}(\varepsilon )$ уравнения (2.6). Действительно, из этих равенств вытекает, что значение $t = {{T}_{\varphi }}(\varepsilon ) - {{\sigma }_{0}}$ асимптотически близко к $t = {{T}_{0}} - {{\sigma }_{0}}$ и в силу условия (2.34) принадлежит отрезку $[{{t}_{0}}(\varepsilon ,z) + 1 + {{\sigma }_{0}},{{T}_{0}} - {{\sigma }_{0}}{\text{/}}2]$. А отсюда и из (2.34) автоматически следует, что корень $t = {{T}_{\varphi }}(\varepsilon )$ допускает при $\varepsilon \to 0$ равномерную по $\varphi \in S$ асимптотику

(2.36)
${{T}_{\varphi }}(\varepsilon ) = {{T}_{0}} - \varepsilon \left[ {z + ln(exp(az) + {{t}_{0}} - {{t}_{ * }}) - \left( {1 + \frac{1}{a}} \right)ln(exp(az) + {{t}_{0}} - {{t}_{ * }} - 1)} \right] + O\left( {{{\varepsilon }^{2}}ln\frac{1}{\varepsilon }} \right).$
Кроме того, из очевидного равенства ${{\dot {\omega }}_{{1,\varphi }}}(t,\varepsilon ) = 1 + O(exp( - q{\text{/}}\varepsilon ))$, имеющего место на отрезке ${{t}_{0}}(\varepsilon ,z) + 1 + {{\sigma }_{0}} \leqslant t \leqslant {{T}_{0}} - {{\sigma }_{0}}{\text{/}}2$, заключаем, что этот корень является простым.

2.3. Завершение доказательства теоремы 1.1

Приступим к реализации описанной в п. 2.1 схемы. В связи с этим обратимся к оператору (2.7), который согласно нашим построениям корректно определен на множестве (2.3). Покажем теперь, что при подходящем выборе констант ${{q}_{j}}$, $j = 1,\; \ldots ,\;4$, и ${{z}_{1}}$, ${{z}_{2}}$ в (2.3), (2.5) оператор $\Pi $ действует из $S$ в $S.$

Действительно, объединяя формулу (1.5) для ${{\omega }_{0}}(t)$ с вытекающими из (2.9) включениями $\left[ {{{T}_{0}} - 1 - {{\sigma }_{0}},{{T}_{0}} - {{\sigma }_{0}}} \right] \subset \left[ {{{t}_{0}} + {{\sigma }_{0}},{{T}_{0}} - {{\sigma }_{0}}} \right]$, $\left[ {{{T}_{0}} - 1 - {{\sigma }_{0}},{{T}_{0}} - {{\sigma }_{0}}} \right] \subset \left[ {{{t}_{{ * * }}} + {{\sigma }_{0}},{{T}_{0}} - {{\sigma }_{0}}} \right]$, нетрудно увидеть, что ${{\omega }_{0}}(t) < 0$, ${{\omega }_{0}}(t - {{t}_{ * }}) < 0$ $\forall t \in \left[ {{{T}_{0}} - 1 - {{\sigma }_{0}},{{T}_{0}} - {{\sigma }_{0}}} \right]$. А отсюда и из асимптотических представлений (2.35), (2.36) следует, что условия

$ - {{q}_{1}} \leqslant {{\omega }_{{1,\varphi }}}(t + {{T}_{\varphi }}(\varepsilon ),\varepsilon ) \leqslant - {{q}_{2}},\quad - {\kern 1pt} {{q}_{3}} \leqslant {{\omega }_{{2,\varphi }}}(t + {{T}_{\varphi }}(\varepsilon ),\varepsilon ) \leqslant - {{q}_{4}}\quad \forall t \in [ - 1 - {{\sigma }_{0}}, - {{\sigma }_{0}}],\quad \forall \varphi \in S$
заведомо выполняются при всех достаточно малых $\varepsilon > 0$ с любыми фиксированными константами

(2.37)
${{q}_{1}} > - {\kern 1pt} \mathop {min}\limits_{{{T}_{0}} - 1 - {{\sigma }_{0}} \leqslant t \leqslant {{T}_{0}} - {{\sigma }_{0}}} {{\omega }_{0}}(t),\quad 0 < {{q}_{2}} < - {\kern 1pt} \mathop {max}\limits_{{{T}_{0}} - 1 - {{\sigma }_{0}} \leqslant t \leqslant {{T}_{0}} - {{\sigma }_{0}}} {{\omega }_{0}}(t),$
(2.38)
${{q}_{3}} > - {\kern 1pt} \mathop {min}\limits_{{{T}_{0}} - 1 - {{\sigma }_{0}} \leqslant t \leqslant {{T}_{0}} - {{\sigma }_{0}}} {{\omega }_{0}}(t - {{t}_{ * }}),\quad 0 < {{q}_{4}} < - {\kern 1pt} \mathop {max}\limits_{{{T}_{0}} - 1 - {{\sigma }_{0}} \leqslant t \leqslant {{T}_{0}} - {{\sigma }_{0}}} {{\omega }_{0}}(t - {{t}_{ * }}).$

Проверим теперь справедливость для функционала (2.4), вычисленного на $\Pi (\varphi )$, включения

(2.39)
${{z}_{{\Pi (\varphi )}}}\;\mathop = \limits^{{\text{def}}} \;\frac{1}{\varepsilon }\left( {{{{\left. {{{\omega }_{{2,\varphi }}}(t,\varepsilon )} \right|}}_{{t = {{T}_{\varphi }}(\varepsilon ) - {{\sigma }_{0}}}}} + {{\sigma }_{0}} + \frac{b}{a} - \frac{\varepsilon }{a}ln\frac{1}{\varepsilon }} \right) \in \Omega \quad \forall \varphi \in S.$
Для этого привлечем асимптотические представления (2.22), (2.36), из которых получаем равномерное по $\varphi \in S$ асимптотическое равенство
(2.40)
${{z}_{{\Pi (\varphi )}}} = F(z) + O\left( {\varepsilon ln\frac{1}{\varepsilon }} \right),\quad \varepsilon \to 0,$
где

(2.41)
$F(z) = \left( {1 + \frac{1}{a}} \right)ln(exp(az) + {{t}_{0}} - {{t}_{ * }} - 1) - ln(exp(az) + {{t}_{0}} - {{t}_{ * }}).$

Введем в рассмотрение одномерное отображение

(2.42)
$z \mapsto F(z),$
где $F(z)$ – функция (2.41). Для его анализа воспользуемся результатами статьи [6]. Напомним, что в [6] изучалось отображение
(2.43)
$y \mapsto {{\psi }_{{{{r}_{1}},{{r}_{2}}}}}(y)\;\mathop = \limits^{{\text{def}}} \;ln({{r}_{1}} + exp( - ay)) - \left( {1 + \frac{1}{a}} \right)ln({{r}_{2}} + exp( - ay))$
с параметрами $a > 1$, ${{r}_{1}},{{r}_{2}} > 0$ и было показано, что при условии
(2.44)
${{r}_{1}} > \left( {1 + \frac{1}{a}} \right){{r}_{2}}$
оно имеет единственную экспоненциально устойчивую неподвижную точку ${{y}_{ * }} = {{y}_{ * }}(a,{{r}_{1}},{{r}_{2}})$.

В нашем случае после замены $y = - z$ отображение (2.42) приводится к виду (2.43) с параметрами ${{r}_{1}} = {{t}_{0}} - {{t}_{ * }}$, ${{r}_{2}} = {{t}_{0}} - {{t}_{ * }} - 1$, удовлетворяющими неравенству (2.44). Тем самым, у него существует единственная устойчивая неподвижная точка ${{z}_{ * }} = {{z}_{ * }}(a,b)$. Добавим еще, что ${{z}_{ * }}(a,b) \to - \infty $ при $b \to 1 - 0$.

Учитывая приведенные факты, в (2.5) положим ${{z}_{1}} = {{z}_{ * }} - \delta $, ${{z}_{2}} = {{z}_{ * }} + \delta $, где $\delta > 0$ настолько мало, что для производной $F{\text{'}}(z)$ имеет место оценка

(2.45)
$\mathop {max}\limits_{{{z}_{ * }} - \delta \leqslant z \leqslant {{z}_{ * }} + \delta } F{\kern 1pt} {\text{'}}(z) < 1.$
В результате, принимая во внимание асимптотическое представление (2.40), убеждаемся в том, что при всех $\varepsilon \ll 1$ требуемое включение (2.39) действительно выполняется.

Итак, мы убедились, что при условиях (2.9), (2.37), (2.38), (2.45), наложенных на параметры ${{\sigma }_{0}}$, $\delta $, ${{q}_{j}}$, $j = 1,\; \ldots ,\;4$, оператор $\Pi $ переводит множество $S$ в себя. Далее, в силу формулы (2.36) и очевидного неравенства ${{T}_{0}} > 1$ имеем ${{T}_{\varphi }}(\varepsilon ) > 1$ $\forall \varphi \in S$, а значит, этот оператор является компактным. Тем самым, согласно принципу Шаудера он допускает в $S$ хотя бы одну неподвижную точку ${{\varphi }_{ * }} = {{\varphi }_{ * }}(t,\varepsilon )$. Как уже было сказано в п. 2.1, соответствующее решение

(2.46)
$\omega (t,\varepsilon ) = {{\left. {{{\omega }_{{{{\varphi }_{ * }}}}}} \right|}_{{\varphi = {{\varphi }_{ * }}}}} = \operatorname{colon} ({{\omega }_{1}}(t,\varepsilon ),{{\omega }_{2}}(t,\varepsilon ))$
системы (2.1) оказывается периодическим с периодом $T(\varepsilon ) = {{\left. {{{T}_{\varphi }}} \right|}_{{\varphi = {{\varphi }_{ * }}}}}$.

Отдельно остановимся на вычислении асимптотики решения (2.46). В связи с этим положим ${{z}_{ * }}(\varepsilon ) = {{z}_{{{{\varphi }_{ * }}}}} = {{z}_{{\Pi ({{\varphi }_{ * }})}}}$. Из асимптотического представления (2.40) очевидным образом следует, что

(2.47)
${{z}_{ * }}(\varepsilon ) = {{z}_{ * }}(a,b) + O\left( {\varepsilon ln\frac{1}{\varepsilon }} \right),\quad \varepsilon \to 0,$
где, напомним, ${{z}_{ * }}(a,b)$ – неподвижная точка отображения (2.42). Что же касается асимптотических представлений для ${{\omega }_{1}}(t,\varepsilon )$, ${{\omega }_{2}}(t,\varepsilon )$, то они получаются из соответствующих асимптотических формул для ${{\omega }_{{1,\varphi }}}(t,\varepsilon )$, ${{\omega }_{{2,\varphi }}}(t,\varepsilon )$ при подстановке вместо $z$ значения (2.47). В частности, для периода $T(\varepsilon )$ решения (2.46) из (2.36), (2.47) имеем

(2.48)
$T(\varepsilon ) = {{T}_{0}} + O\left( {{{\varepsilon }^{2}}ln\frac{1}{\varepsilon }} \right),\quad \varepsilon \to 0.$

Проделанные в данном пункте построения вместе с содержащимся в п. 2.2 асимптотическим анализом позволяют разобраться с нашей главной проблемой – обоснованием теоремы 1.1. Для этого рассмотрим корень $t = \tilde {t}(\varepsilon )$ уравнения ${{\omega }_{1}}(t,\varepsilon ) = 0$, асимптотически близкий к нулю. Из представления для ${{\omega }_{{1,\varphi }}}(t,\varepsilon )$ (см. (2.23)) и из равенства $\mathop {\dot {\omega }}\nolimits_{1,\varphi } (t,\varepsilon ) = 1 + O(exp( - q{\text{/}}\varepsilon ))$, имеющего место на отрезке $ - {{\sigma }_{0}} \leqslant t \leqslant 1 - {{\sigma }_{0}}$, вытекает, что этот корень является простым и допускает асимптотику

(2.49)
$\tilde {t}(\varepsilon ) = O\left( {exp\left( { - \frac{q}{\varepsilon }} \right)} \right),\quad \varepsilon \to 0.$
Полагая затем
(2.50)
${{u}_{{j, * }}}(t,\lambda ) = exp(\lambda {{\omega }_{{j, * }}}(t,\lambda )),\quad {{\omega }_{{j, * }}}(t,\lambda ) = {{\omega }_{j}}{{\left. {(t + \tilde {t}(\varepsilon ),\varepsilon )} \right|}_{{\varepsilon = 1/\lambda }}},\quad j = 1,2,$
получаем пару циклов (1.10) системы (1.6), (1.8) периода

(2.51)
${{T}_{ * }}(\lambda ) = {{\left. {T(\varepsilon )} \right|}_{{\varepsilon = 1/\lambda }}}.$

В заключение отметим, что для функций ${{\omega }_{{j, * }}}(t,\lambda )$, $j = 1,2$ и ${{T}_{ * }}(\lambda )$ из (2.50), (2.51) справедливы асимптотические представления (1.12). Действительно, нужная формула для ${{T}_{ * }}(\lambda )$ очевидным образом следует из (2.48), (2.51), а асимптотические формулы для ${{\omega }_{{j, * }}}(t,\lambda )$, $j = 1,2$ – следствия соотношений (2.35), (2.47)–(2.51). Теорема 1.1 полностью доказана.

3. ДОКАЗАТЕЛЬСТВО ТЕОРЕМЫ 1.2

3.1. Общий план исследования

Как уже отмечалось в разд. 1, свойства устойчивости пары циклов (1.10) одинаковы. Поэтому обратимся к первому из них и заметим, что проблема его устойчивости сводится к исследованию устойчивости цикла (2.46) системы (2.1). В свою очередь, за устойчивость цикла (2.46) отвечает расположение мультипликаторов линейной системы в вариациях

(3.1)
$\dot {g} = A(t,\varepsilon )g + B(t,\varepsilon )g(t - 1),$
где $g = {\text{colon}}({{g}_{1}},{{g}_{2}})$,

(3.2)
$A(t,\varepsilon ) = \left( {\begin{array}{*{20}{c}} { - {{A}_{1}}(t,\varepsilon )}&{{{A}_{1}}(t,\varepsilon )} \\ {{{A}_{2}}(t,\varepsilon )}&{ - {{A}_{2}}(t,\varepsilon )} \end{array}} \right),\quad B(t,\varepsilon ) = {\text{diag}}({{B}_{1}}(t,\varepsilon ),{{B}_{2}}(t,\varepsilon )),$
(3.3)
${{A}_{1}}(t,\varepsilon ) = \frac{1}{\varepsilon }exp\left( {\frac{{{{\omega }_{2}}(t,\varepsilon ) - {{\omega }_{1}}(t,\varepsilon ) - b}}{\varepsilon }} \right),\quad {{A}_{2}}(t,\varepsilon ) = \frac{1}{\varepsilon }exp\left( {\frac{{{{\omega }_{1}}(t,\varepsilon ) - {{\omega }_{2}}(t,\varepsilon ) - b}}{\varepsilon }} \right),$
(3.4)
${{B}_{j}}(t,\varepsilon ) = \frac{1}{\varepsilon }f{\kern 1pt} {\text{'}}\left( {exp\left( {\frac{{{{\omega }_{j}}(t - 1,\varepsilon )}}{\varepsilon }} \right)} \right)exp\left( {\frac{{{{\omega }_{j}}(t - 1,\varepsilon )}}{\varepsilon }} \right),\quad j = 1,2.$

Поясним смысл термина “мультипликатор” применительно к системе (3.1). В связи с этим обратимся к введенному в п. 2.1 пространству $E$ с нормой (2.2) и в этом пространстве определим так называемый оператор монодромии $V(\varepsilon ),$ являющийся линейным и действующий на произвольную вектор-функцию ${{g}_{0}}(t) = {\text{colon}}({{g}_{{1,0}}}(t),{{g}_{{2,0}}}(t)) \in E$ по правилу

(3.5)
$V(\varepsilon ){{g}_{0}} = g(t + T(\varepsilon ),{{g}_{0}},\varepsilon ),\quad - {\kern 1pt} 1 - {{\sigma }_{0}} \leqslant t \leqslant - {{\sigma }_{0}}.$
Здесь $g(t,{{g}_{0}},\varepsilon ) = \operatorname{colon} ({{g}_{1}}(t,{{g}_{0}},\varepsilon ),{{g}_{2}}(t,{{g}_{0}},\varepsilon ))$ – решение системы (3.1) на полуоси $t \geqslant - {{\sigma }_{0}}$ с начальной функцией ${{g}_{0}}(t)$, $ - 1 - {{\sigma }_{0}} \leqslant t \leqslant - {{\sigma }_{0}}$, а $T(\varepsilon )$ – период цикла (2.46). Отметим, что в силу вытекающего из (2.48) неравенства $T(\varepsilon ) > 1$ оператор (3.5) является компактным. Что же касается мультипликаторов системы (3.1), то таковыми по аналогии со случаем обыкновенных дифференциальных уравнений будем называть собственные значения оператора $V(\varepsilon )$. Занумеруем их в порядке убывания модулей и обозначим через ${{\nu }_{s}}(\varepsilon ) \in \mathbb{C}$, $s \in \mathbb{N}$.

В следующих двух пунктах мы проведем асимптотическое вычисление мультипликаторов ${{\nu }_{s}}(\varepsilon )$, $s \geqslant 1$. На этом пути будут получены соотношения вида

(3.6)
${{\nu }_{1}}(\varepsilon ) \equiv 1,\quad {{\nu }_{2}}(\varepsilon ) = {{\left. {F{\kern 1pt} {\text{'}}(z)} \right|}_{{z = {{z}_{ * }}(a,b)}}} + O\left( {\varepsilon ln\frac{1}{\varepsilon }} \right),\quad \mathop {sup}\limits_{s \geqslant 3} \left| {{{\nu }_{s}}(\varepsilon )} \right| \leqslant exp\left( { - \frac{q}{\varepsilon }} \right),\quad M = {\text{const}} > 0,$
означающие, что цикл (2.46) экспоненциально орбитально устойчив (в метрике фазового пространства $E$).

Спектральный анализ оператора (3.5) базируется на представлении

(3.7)
$V(\varepsilon ) = {{V}_{1}}(\varepsilon ) + {{V}_{2}}(\varepsilon ),$
где операторы ${{V}_{j}}(\varepsilon ):E \to E$, $j = 1,2$, задаются равенствами
(3.8)
${{V}_{1}}(\varepsilon ){{g}_{0}} = g(t + T(\varepsilon ),{{\tilde {g}}_{0}},\varepsilon ),\quad - {\kern 1pt} 1 - {{\sigma }_{0}} \leqslant t \leqslant - {{\sigma }_{0}},$
(3.9)
${{V}_{2}}(\varepsilon ){{g}_{0}} = {{g}_{{1,0}}}( - {{\sigma }_{0}}){{g}_{ * }}(t + T(\varepsilon ),\varepsilon ) + {{g}_{{2,0}}}( - {{\sigma }_{0}}){{g}_{{ * * }}}(t + T(\varepsilon ),\varepsilon ),\quad - {\kern 1pt} 1 - {{\sigma }_{0}} \leqslant t \leqslant - {{\sigma }_{0}}.$
Здесь ${{\tilde {g}}_{0}}(t) = {\text{colon}}({{g}_{{1,0}}}(t) - {{g}_{{1,0}}}( - {{\sigma }_{0}}),{{g}_{{2,0}}}(t) - {{g}_{{2,0}}}( - {{\sigma }_{0}}))$, а через
(3.10)
${{g}_{ * }}(t,\varepsilon ) = {\text{colon}}({{g}_{{1, * }}}(t,\varepsilon ),{{g}_{{2, * }}}(t,\varepsilon )),\quad {{g}_{{ * * }}}(t,\varepsilon ) = {\text{colon}}({{g}_{{1, * * }}}(t,\varepsilon ),{{g}_{{2, * * }}}(t,\varepsilon ))$
обозначены специальные решения системы (3.1) с начальными функциями ${{g}_{0}}(t) \equiv {\text{colon}}(1,0)$, $ - 1 - {{\sigma }_{0}} \leqslant t \leqslant - {{\sigma }_{0}}$, и ${{g}_{0}}(t) \equiv {\text{colon}}(0,1)$, $ - 1 - {{\sigma }_{0}} \leqslant t \leqslant - {{\sigma }_{0}}$ соответственно. В последующем слагаемые ${{V}_{1}}(\varepsilon )$ и ${{V}_{2}}(\varepsilon )$ из (3.7) будем называть бесконечномерной и конечномерной частями оператора монодромии $V(\varepsilon )$.

Ниже мы убедимся, что норма оператора (3.8) экспоненциально мала, т.е. имеет порядок $O(exp( - q{\text{/}}\varepsilon ))$. Что же касается оператора (3.9), то, поскольку он является конечномерным, его спектр состоит из собственного числа $\nu = 0$ бесконечной кратности и из собственных значений матрицы

(3.11)
$\mathcal{D}(\varepsilon ) = \left( {\begin{array}{*{20}{c}} {{{g}_{{1, * }}}(T(\varepsilon ) - {{\sigma }_{0}},\varepsilon )}&{{{g}_{{1, * * }}}(T(\varepsilon ) - {{\sigma }_{0}},\varepsilon )} \\ {{{g}_{{2, * }}}(T(\varepsilon ) - {{\sigma }_{0}},\varepsilon )}&{{{g}_{{2, * * }}}(T(\varepsilon ) - {{\sigma }_{0}},\varepsilon )} \end{array}} \right).$
Таким образом, проблема обоснования соотношений (3.6) сводится в конечном итоге к асимптотическому вычислению спектра матрицы (3.11).

3.2. Анализ бесконечномерной части оператора монодромии

Будем считать, что начальная функция ${{g}_{0}}(t)$ в формуле (3.5) удовлетворяет условию $\left\| {{{g}_{0}}} \right\| \leqslant 1$, где $\left\| {\, * \,} \right\|$ – норма (2.2). Тогда функция ${{\tilde {g}}_{0}}(t)$ из (3.8) принадлежит множеству

(3.12)
$B = \left\{ {{{g}_{0}}(t) \in E:{{g}_{{1,0}}}( - {{\sigma }_{0}}) = {{g}_{{2,0}}}( - {{\sigma }_{0}}) = 0,\;\left\| {{{g}_{0}}} \right\| \leqslant 2} \right\}.$
Тем самым, для анализа оператора (3.8) необходимо выявить свойства решения $g(t,{{g}_{0}},\varepsilon )$ системы (3.1) с произвольным начальным условием ${{g}_{0}}(t)$ из множества (3.12). А именно, ниже нам потребуется следующая

Лемма 3.1. Найдутся такие положительные константы $M$, $q$ и такое достаточно малое ${{\varepsilon }_{0}} > 0$, что при всех $0 < \varepsilon \leqslant {{\varepsilon }_{0}}$ решение $g(t,{{g}_{0}},\varepsilon )$ системы (3.1) с произвольным начальным условием ${{g}_{0}}(t) \in B$ допускает оценку

(3.13)
$\mathop {max}\limits_{ - {{\sigma }_{0}} \leqslant t \leqslant T(\varepsilon ) - {{\sigma }_{0}}} {{\left\| {g(t,{{g}_{0}},\varepsilon )} \right\|}_{{{{\mathbb{R}}^{2}}}}} \leqslant Mexp\left( { - \frac{q}{\varepsilon }} \right),$
где ${{\left\| g \right\|}_{{{{\mathbb{R}}^{2}}}}} = \mathop {max}\limits_{j = 1,2} \left| {{{g}_{j}}} \right|$ $\forall g = {\text{colon}}({{g}_{1}},{{g}_{2}}) \in {{\mathbb{R}}^{2}}$.

Доказательство. Приведем сначала некоторые свойства коэффициентов (3.3), (3.4) системы (3.1). Из установленных в разд. 2 асимптотических представлений для компонент ${{\omega }_{1}}(t,\varepsilon )$, ${{\omega }_{2}}(t,\varepsilon )$ цикла (2.46) вытекают оценки вида

(3.14)
$0 < {{A}_{j}}(t,\varepsilon ) \leqslant \frac{{{{M}_{j}}}}{\varepsilon },\quad j = 1,2,\quad \left| {{{B}_{1}}(t,\varepsilon )} \right| \leqslant \frac{{{{M}_{3}}}}{\varepsilon },\quad \left| {{{B}_{2}}(t,\varepsilon )} \right| \leqslant \frac{{{{M}_{4}}}}{\varepsilon }\quad \forall t \in \left[ { - {{\sigma }_{0}},T(\varepsilon ) - {{\sigma }_{0}}} \right];$
(3.15)
$\mathop {max}\limits_{ - {{\sigma }_{0}} \leqslant t \leqslant 1 - {{\sigma }_{0}}} (\left| {{{B}_{1}}(t,\varepsilon )} \right| + \left| {{{B}_{2}}(t,\varepsilon )} \right|) \leqslant {{M}_{5}}exp\left( { - \frac{q}{\varepsilon }} \right),$
где ${{M}_{j}} = {\text{const}} > 0$, $j = 1,\; \ldots ,\;5$.

Обоснование неравенства (3.13) начнем с отрезка $ - {{\sigma }_{0}} \leqslant t \leqslant 1 - {{\sigma }_{0}}$, на котором согласно (3.12) интересующее нас решение $g(t,{{g}_{0}},\varepsilon )$ определяется из задачи Коши

(3.16)
$\dot {g} = A(t,\varepsilon )g + G(t,{{g}_{0}},\varepsilon ),\quad {{\left. g \right|}_{{t = - {{\sigma }_{0}}}}} = 0,$
где $G(t,{{g}_{0}},\varepsilon ) = B(t,\varepsilon ){{g}_{0}}(t - 1)$. Анализ этой задачи опирается на следующие два факта.

Во-первых, из явного вида неоднородности $G(t,{{g}_{0}},\varepsilon )$ и оценок (3.15) имеем

(3.17)
$\mathop {max}\limits_{ - {{\sigma }_{0}} \leqslant t \leqslant 1 - {{\sigma }_{0}}} {{\left\| {G(t,{{g}_{0}},\varepsilon )} \right\|}_{{{{\mathbb{R}}^{2}}}}} \leqslant Mexp\left( { - \frac{q}{\varepsilon }} \right),\quad M = {\text{const}} > 0.$

Во-вторых, матрица Коши $K(t,\tau ,\varepsilon )$ системы

(3.18)
$\dot {g} = A(t,\varepsilon )g$
допускает оценку

(3.19)
$\mathop {max}\limits_{ - {{\sigma }_{0}} \leqslant \tau \leqslant t \leqslant T(\varepsilon ) - {{\sigma }_{0}}} {{\left\| {K(t,\tau ,\varepsilon )} \right\|}_{{{{\mathbb{R}}^{2}} \to {{\mathbb{R}}^{2}}}}} \leqslant \frac{M}{\varepsilon },\quad M = {\text{const}} > 0.$

Действительно, из явного вида матрицы $A(t,\varepsilon )$ (см. (3.2)) следует, что после выполнения замен ${{h}_{1}} = {{g}_{1}} - {{g}_{2}}$, ${{h}_{2}} = {{g}_{1}} + {{g}_{2}}$ система (3.18) приобретает треугольный вид

(3.20)
${{\dot {h}}_{1}} = - ({{A}_{1}}(t,\varepsilon ) + {{A}_{2}}(t,\varepsilon )){{h}_{1}},\quad {{\dot {h}}_{2}} = ({{A}_{2}}(t,\varepsilon ) - {{A}_{1}}(t,\varepsilon )){{h}_{1}}.$
Тем самым, матрица Коши $\tilde {K}(t,\tau ,\varepsilon )$ этой системы задается явными формулами
$\tilde {K}(t,\tau ,\varepsilon ) = \left( {\begin{array}{*{20}{c}} {{{\mathcal{K}}_{{1,1}}}(t,\tau ,\varepsilon )}&{{{\mathcal{K}}_{{1,2}}}(t,\tau ,\varepsilon )} \\ {{{\mathcal{K}}_{{2,1}}}(t,\tau ,\varepsilon )}&{{{\mathcal{K}}_{{2,2}}}(t,\tau ,\varepsilon )} \end{array}} \right),$
$\begin{gathered} {{\mathcal{K}}_{{1,1}}}(t,\tau ,\varepsilon ) = exp\left( { - \int\limits_\tau ^t {({{A}_{1}}(s,\varepsilon ) + {{A}_{2}}(s,\varepsilon ))} ds} \right),\quad {{\mathcal{K}}_{{1,2}}}(t,\tau ,\varepsilon ) \equiv 0, \\ {{\mathcal{K}}_{{2,1}}}(t,\tau ,\varepsilon ) = \int\limits_\tau ^t {({{A}_{2}}(s,\varepsilon ) - {{A}_{1}}(s,\varepsilon ))} {{\mathcal{K}}_{{1,1}}}(s,\tau ,\varepsilon )ds,\quad {{\mathcal{K}}_{{2,2}}}(t,\tau ,\varepsilon ) \equiv 1. \\ \end{gathered} $
А отсюда и из (3.14) заключаем, что для $\tilde {K}(t,\tau ,\varepsilon )$ аналогичная (3.19) оценка выполняется. Осуществляя затем обратный переход от системы (3.20) к (3.18), убеждаемся в справедливости оценки (3.19) и для исходной матрицы $K(t,\tau ,\varepsilon ).$

Приведенная информация позволяет уже достаточно просто разобраться с задачей Коши (3.16). Для того чтобы сделать это, учтем оценки (3.17), (3.19) в явной формуле

(3.21)
$g(t,{{g}_{0}},\varepsilon ) = \int\limits_{ - {{\sigma }_{0}}}^t {K(t,\tau ,\varepsilon )} G(\tau ,{{g}_{0}},\varepsilon )d\tau ,\quad - {\kern 1pt} {{\sigma }_{0}} \leqslant t \leqslant 1 - {{\sigma }_{0}},$
для ее решения $g(t,{{g}_{0}},\varepsilon )$. В результате убеждаемся в справедливости на промежутке времени $ - {{\sigma }_{0}} \leqslant t \leqslant 1 - {{\sigma }_{0}}$ неравенства вида

(3.22)
$\mathop {max}\limits_t {{\left\| {g(t,{{g}_{0}},\varepsilon )} \right\|}_{{{{\mathbb{R}}^{2}}}}} \leqslant Mexp\left( { - \frac{q}{\varepsilon }} \right),\quad M = {\text{const}} > 0.$

Для распространения оценки (3.22) на отрезок $1 - {{\sigma }_{0}} \leqslant t \leqslant T(\varepsilon ) - {{\sigma }_{0}}$ воспользуемся методом шагов, т.е. разобьем указанный отрезок на промежутки времени длины не более единицы и будем последовательно рассматривать получившиеся части.

Общая схема действий здесь такова. Предположим, что оценка (3.22) уже установлена на отрезке вида $\bar {t} - 1 \leqslant t \leqslant \bar {t}$, где $\bar {t} \in [1 - {{\sigma }_{0}},T(\varepsilon ) - {{\sigma }_{0}}]$. Тогда для отыскания $g(t,{{g}_{0}},\varepsilon )$ при $\bar {t} \leqslant t \leqslant \bar {t} + 1$ получаем аналогичную (3.16) задачу Коши

(3.23)
$\dot {g} = A(t,\varepsilon )g + G(t,{{g}_{0}},\varepsilon ),\quad {{\left. g \right|}_{{t = \bar {t}}}} = g(\bar {t},{{g}_{0}},\varepsilon )$
с неоднородностью $G(t,{{g}_{0}},\varepsilon ) = B(t,\varepsilon )g(t - 1,{{g}_{0}},\varepsilon )$. Далее, в силу свойств (3.14) и установленного на предыдущем шаге представления
${{\left\| {g(t - 1,{{g}_{0}},\varepsilon )} \right\|}_{{{{\mathbb{R}}^{2}}}}} = O\left( {exp\left( { - \frac{q}{\varepsilon }} \right)} \right),\quad \varepsilon \to 0,\quad \bar {t} \leqslant t \leqslant \bar {t} + 1,$
имеем
${{\left\| {g(\bar {t},{{g}_{0}},\varepsilon )} \right\|}_{{{{\mathbb{R}}^{2}}}}} \leqslant Mexp\left( { - \frac{q}{\varepsilon }} \right),\quad \mathop {max}\limits_{\bar {t} \leqslant t \leqslant \bar {t} + 1} {{\left\| {G(t,{{g}_{0}},\varepsilon )} \right\|}_{{{{\mathbb{R}}^{2}}}}} \leqslant Mexp\left( { - \frac{q}{\varepsilon }} \right),$
где $M = {\text{const}} > 0$. Объединяя затем эту информацию с неравенством (3.19) и привлекая аналогичную (3.21) явную формулу
$g(t,{{g}_{0}},\varepsilon ) = K(t,\bar {t},\varepsilon )g(\bar {t},{{g}_{0}},\varepsilon ) + \int\limits_{\bar {t}}^t {K(t,\tau ,\varepsilon )} G(\tau ,{{g}_{0}},\varepsilon )d\tau ,\quad \bar {t} \leqslant t \leqslant \bar {t} + 1,$
для решения задачи (3.23), распространяем оценку (3.22) на шаг вперед. Ясно, что в итоге требуемое неравенство будет обосновано на всем промежутке времени $ - {{\sigma }_{0}} \leqslant t \leqslant T(\varepsilon ) - {{\sigma }_{0}}$. Лемма 3.1 доказана.

Завершая рассмотрение бесконечномерной части оператора монодромии, т.е. оператора (3.8), отметим, что в силу (3.13) для него справедлива оценка

(3.24)
${{\left\| {{{V}_{1}}(\varepsilon )} \right\|}_{{E \to E}}} \leqslant Mexp\left( { - \frac{q}{\varepsilon }} \right),\quad M = {\text{const}} > 0.$

3.3. Исследование конечномерной части оператора монодромии

Для анализа конечномерного оператора (3.9) в первую очередь необходимо разобраться с асимптотикой двух введенных выше специальных решений (3.10) системы (3.1).

Начнем с рассмотрения вектор-функции ${{g}_{{ * * }}}(t,\varepsilon )$. Для этого нам потребуется специальное решение

(3.25)
$\omega (t,\varepsilon ,z) = {\text{colon}}({{\omega }_{1}}(t,\varepsilon ,z),{{\omega }_{2}}(t,\varepsilon ,z)),\quad t \geqslant - {{\sigma }_{0}},$
системы (2.1) с начальным условием
$\varphi (t,\varepsilon ,z) \equiv {\text{colon}}\left( { - {{\sigma }_{0}}, - {{\sigma }_{0}} - \frac{b}{a} + \frac{\varepsilon }{a}ln\frac{1}{\varepsilon } + \varepsilon z} \right),\quad - {\kern 1pt} 1 - {{\sigma }_{0}} \leqslant t \leqslant - {{\sigma }_{0}},$
где свободный параметр $z$ пробегает множество $\Omega $ (см. (2.5)). Поскольку при соответствующем выборе констант ${{q}_{j}}$, $j = 1,\; \ldots ,\;4$, в (2.3) справедливо включение $\varphi (t,\varepsilon ,z) \in S$ $\forall z \in \Omega $, то асимптотика вектор-функции (3.25) нам известна (см. п. 2.2). Более того, нетрудно убедиться (соответствующие вполне понятные выкладки опустим), что на каждом из рассмотренных в п. 2.2 промежутков времени асимптотика вектор-функции
(3.26)
$h(t,\varepsilon ,z) = {\text{colon}}({{h}_{1}}(t,\varepsilon ,z),{{h}_{2}}(t,\varepsilon ,z))\;\mathop = \limits^{{\text{def}}} \;\frac{1}{\varepsilon }\frac{{\partial \omega }}{{\partial z}}(t,\varepsilon ,z)$
получается из соответствующих формул для $\omega (t,\varepsilon ,z)$ при дифференцировании по $z$ и делении на $\varepsilon $ (с сохранением равномерных по $t$, $z$ остатков). Далее, введем в рассмотрение вектор-функцию ${{h}_{{ * * }}}(t,\varepsilon ) = {{\left. {h(t,\varepsilon ,z)} \right|}_{{z = {{z}_{ * }}(\varepsilon )}}}$, где ${{z}_{ * }}(\varepsilon )$ – значение функционала  (2.4) на периодическом решении (2.46) (см. (2.47)). Справедливо следующее утверждение.

Лемма 3.2. При $\varepsilon \to 0$ равномерно по $t \in \left[ { - {{\sigma }_{0}},T(\varepsilon ) - {{\sigma }_{0}}} \right]$ выполняется асимптотическое представление

(3.27)
${{g}_{{ * * }}}(t,\varepsilon ) = {{h}_{{ * * }}}(t,\varepsilon ) + O\left( {exp\left( { - \frac{q}{\varepsilon }} \right)} \right).$

Доказательство. Обратим внимание на следующие два обстоятельства. Во-первых, вектор-функция (3.26) по своему определению является решением задачи Коши

(3.28)
$\dot {h} = A(t,\varepsilon ,z)h + B(t,\varepsilon ,z)h(t - 1),\quad h \equiv {\text{colon}}(0,1)\quad {\text{при}}\quad - {\kern 1pt} 1 - {{\sigma }_{0}} \leqslant t \leqslant - {{\sigma }_{0}},$
где матрицы $A(t,\varepsilon ,z)$, $B(t,\varepsilon ,z)$ задаются аналогичными (3.2)–(3.4) равенствами, в которых ${{\omega }_{1}}(t,\varepsilon )$, ${{\omega }_{2}}(t,\varepsilon )$ следует заменить на компоненты ${{\omega }_{1}}(t,\varepsilon ,z)$, ${{\omega }_{2}}(t,\varepsilon ,z)$ вектор-функции (3.25). Во-вторых, на отрезке $ - {{\sigma }_{0}} \leqslant t \leqslant T(\varepsilon ) - {{\sigma }_{0}}$ справедливы представления
(3.29)
$A(t,\varepsilon ) = \tilde {A}(t,\varepsilon ) + {{\Delta }_{1}}(t,\varepsilon ),\quad B(t,\varepsilon ) = \tilde {B}(t,\varepsilon ) + {{\Delta }_{2}}(t,\varepsilon ),$
где $\tilde {A}(t,\varepsilon ) = {{\left. {A(t,\varepsilon ,z)} \right|}_{{z = {{z}_{ * }}(\varepsilon )}}}$, $\tilde {B}(t,\varepsilon ) = {{\left. {B(t,\varepsilon ,z)} \right|}_{{z = {{z}_{ * }}(\varepsilon )}}}$, а остатки ${{\Delta }_{1}}$, ${{\Delta }_{2}}$ таковы, что

(3.30)
$\mathop {max}\limits_{ - {{\sigma }_{0}} \leqslant t \leqslant T(\varepsilon ) - {{\sigma }_{0}}} {{\left\| {{{\Delta }_{j}}(t,\varepsilon )} \right\|}_{{{{\mathbb{R}}^{2}} \to {{\mathbb{R}}^{2}}}}} \leqslant Mexp\left( { - \frac{q}{\varepsilon }} \right),\quad M = {\text{const}} > 0,\quad j = 1,2.$

Действительно, разность $\omega (t,\varepsilon ) - {{\left. {\omega (t,\varepsilon ,z)} \right|}_{{z = {{z}_{ * }}(\varepsilon )}}}$ в первом приближении удовлетворяет при $t \geqslant - {{\sigma }_{0}}$ линейной системе (3.1) с начальным условием при $ - {\kern 1pt} 1 - {{\sigma }_{0}} \leqslant t \leqslant - {{\sigma }_{0}}$, принадлежащим множеству вида (3.12) (с заменой неравенства $\left\| {{{g}_{0}}} \right\| \leqslant 2$ на $\left\| {{{g}_{0}}} \right\| \leqslant M$, $M = {\text{const}} > 0$). Тем самым, опираясь на лемму 3.1, заключаем, что при $\varepsilon \to 0$ равномерно по $t \in \left[ { - 1 - {{\sigma }_{0}}, - {{\sigma }_{0}}} \right]$

(3.31)
${{\omega }_{j}}(t,\varepsilon ) = {{\left. {{{\omega }_{j}}(t,\varepsilon ,z)} \right|}_{{z = {{z}_{ * }}(\varepsilon )}}} + O\left( {exp\left( { - \frac{q}{\varepsilon }} \right)} \right),\quad j = 1,2.$
Далее, из соотношений (3.31) и явного вида матрицы $A(t,\varepsilon )$ (см. (3.2), (3.3)) первая формула (3.29) получается автоматически. Для обоснования же второй из этих формул воспользуемся вытекающей из (1.2) оценкой
$\begin{gathered} \left| {{{u}_{1}}f{\kern 1pt} {\text{'}}({{u}_{1}}) - {{u}_{2}}f{\kern 1pt} {\text{'}}({{u}_{2}})} \right| \leqslant \frac{M}{{1 + min(u_{1}^{2},u_{2}^{2})}}\left| {{{u}_{1}} - {{u}_{2}}} \right|, \\ M = \mathop {\sup }\limits_{u \geqslant 0} (1 + {{u}^{2}})\left| {f{\kern 1pt} {\text{'}}(u) + uf{\kern 1pt} {\text{''}}(u)} \right| < \infty \quad \forall {{u}_{1}},{{u}_{2}} \geqslant 0. \\ \end{gathered} $
Объединяя данную оценку с равенствами (3.4), (3.31), убеждаемся в справедливости нужного представления из (3.29) для $B(t,\varepsilon )$.

Приступим теперь непосредственно к доказательству формулы (3.27). С этой целью подставим в (3.1) выражение $g = {{h}_{{ * * }}}(t,\varepsilon ) + \Delta $. В результате с учетом соотношений (3.29) и того факта, что $h = {{h}_{{ * * }}}(t,\varepsilon )$ является решением задачи Коши (3.28) при $z = {{z}_{ * }}(\varepsilon )$, для отыскания $\Delta $ приходим к задаче

(3.32)
$\dot {\Delta } = A(t,\varepsilon )\Delta + B(t,\varepsilon )\Delta (t - 1) + G(t,\varepsilon ),\quad \Delta \equiv 0\quad {\text{при}}\quad - {\kern 1pt} 1 - {{\sigma }_{0}} \leqslant t \leqslant - {{\sigma }_{0}}$
с неоднородностью

(3.33)
$G(t,\varepsilon ) = {{\Delta }_{1}}(t,\varepsilon ){{h}_{{ * * }}}(t,\varepsilon ) + {{\Delta }_{2}}(t,\varepsilon ){{h}_{{ * * }}}(t - 1,\varepsilon ).$

Отдельно остановимся на свойствах вектор-функции (3.33). Заметим, что из описанного выше способа нахождения асимптотики вектор-функции (3.26) вытекает неравенство

(3.34)
$\mathop {max}\limits_{ - {{\sigma }_{0}} \leqslant t \leqslant T(\varepsilon ) - {{\sigma }_{0}}} {{\left\| {{{h}_{{ * * }}}(t,\varepsilon )} \right\|}_{{{{\mathbb{R}}^{2}}}}} \leqslant M,\quad M = {\text{const}} > 0.$
Применяя, далее, оценки (3.30), (3.34) к (3.33), убеждаемся в том, что

(3.35)
$\mathop {max}\limits_{ - {{\sigma }_{0}} \leqslant t \leqslant T(\varepsilon ) - {{\sigma }_{0}}} {{\left\| {G(t,\varepsilon )} \right\|}_{{{{\mathbb{R}}^{2}}}}} \leqslant Mexp\left( { - \frac{q}{\varepsilon }} \right),\quad M = {\text{const}} > 0.$

Построение решения задачи Коши (3.32) на отрезке $ - {{\sigma }_{0}} \leqslant t \leqslant T(\varepsilon ) - {{\sigma }_{0}}$ проводится методом шагов, причем в силу (3.35) на каждом шаге имеем дело с аналогичной (3.23) линейной неоднородной задачей Коши вида

$\dot {\Delta } = A(t,\varepsilon )\Delta + {{G}_{0}}(t,\varepsilon ),\quad {{\left. \Delta \right|}_{{t = \bar {t}}}} = {{\Delta }_{0}}(\varepsilon ),\quad \bar {t} \leqslant t \leqslant \bar {t} + 1,$
где
$\bar {t} \in \left[ { - {{\sigma }_{0}},T(\varepsilon ) - {{\sigma }_{0}}} \right],\quad \mathop {max}\limits_{\bar {t} \leqslant t \leqslant \bar {t} + 1} {{\left\| {{{G}_{0}}(t,\varepsilon )} \right\|}_{{{{\mathbb{R}}^{2}}}}} + {{\left\| {{{\Delta }_{0}}(\varepsilon )} \right\|}_{{{{\mathbb{R}}^{2}}}}} \leqslant Mexp\left( { - \frac{q}{\varepsilon }} \right),\quad M = {\text{const}} > 0.$
Тем самым, рассуждая как и при обосновании леммы 3.1, приходим к выводу, что
(3.36)
$\mathop {max}\limits_{ - {{\sigma }_{0}} \leqslant t \leqslant T(\varepsilon ) - {{\sigma }_{0}}} {{\left\| \Delta \right\|}_{{{{\mathbb{R}}^{2}}}}} \leqslant Mexp\left( { - \frac{q}{\varepsilon }} \right),\quad M = {\text{const}} > 0.$
Лемма 3.2 доказана.

Для того чтобы изучить асимптотику второго из решений (3.10), рассмотрим вектор-функцию

(3.37)
${{h}_{ * }}(t,\varepsilon ) = \frac{{\dot {\omega }(t,\varepsilon )}}{{{{{\dot {\omega }}}_{1}}( - {{\sigma }_{0}},\varepsilon )}} - \frac{{{{{\dot {\omega }}}_{2}}( - {{\sigma }_{0}},\varepsilon )}}{{{{{\dot {\omega }}}_{1}}( - {{\sigma }_{0}},\varepsilon )}}{{h}_{{ * * }}}(t,\varepsilon ),\quad - {{\sigma }_{0}} \leqslant t \leqslant T(\varepsilon ) - {{\sigma }_{0}},$
где $\omega (t,\varepsilon )$ – периодическое решение (2.46), и заметим, что ее асимптотика нам известна. Действительно, об асимптотике ${{h}_{{ * * }}}(t,\varepsilon )$ уже говорилось выше. В случае же $\dot {\omega }(t,\varepsilon )$ ситуация следующая.

Нетрудно убедиться, что на всех рассмотренных в п. 2.2 промежутках времени асимптотика производных ${{\dot {\omega }}_{{1,\varphi }}}(t,\varepsilon )$, ${{\dot {\omega }}_{{2,\varphi }}}(t,\varepsilon )$ получается при дифференцировании по $t$ соответствующих формул для ${{\omega }_{{1,\varphi }}}(t,\varepsilon )$, ${{\omega }_{{2,\varphi }}}(t,\varepsilon )$ с сохранением всех равномерных по $t$, $\varphi $ остатков. Для обоснования этого факта достаточно на каждом этапе подставить асимптотические представления для ${{\omega }_{{1,\varphi }}}(t,\varepsilon )$, ${{\omega }_{{2,\varphi }}}(t,\varepsilon )$ в правые части системы (2.1) и провести переразложения по $\varepsilon $. Что же касается вектор-функции $\dot {\omega }(t,\varepsilon )$, то асимптотика ее компонент ${{\dot {\omega }}_{1}}(t,\varepsilon )$, ${{\dot {\omega }}_{2}}(t,\varepsilon )$ получается из асимптотических формул для ${{\dot {\omega }}_{{1,\varphi }}}(t,\varepsilon )$, ${{\dot {\omega }}_{{2,\varphi }}}(t,\varepsilon )$ при $z = {{z}_{ * }}(\varepsilon )$, $\varphi = {{\varphi }_{ * }}(t,\varepsilon )$, где, напомним, ${{\varphi }_{ * }}(t,\varepsilon )$ – начальное условие цикла (2.46).

Как оказывается, вектор-функция (3.37) с точностью до экспоненциально малой по норме добавки совпадает с интересующим нас решением ${{g}_{ * }}(t,\varepsilon )$. Точнее говоря, справедлива следующая

Леммя 3.3. При $\varepsilon \to 0$ имеет место равномерное по $t \in \left[ { - {{\sigma }_{0}},T(\varepsilon ) - {{\sigma }_{0}}} \right]$ асимптотическое представление

(3.38)
${{g}_{ * }}(t,\varepsilon ) = {{h}_{ * }}(t,\varepsilon ) + O\left( {exp\left( { - \frac{q}{\varepsilon }} \right)} \right).$

Доказательство. Как и при обосновании леммы 3.2, подставим в систему (3.1) выражение $g = {{h}_{ * }}(t,\varepsilon ) + \Delta $ и учтем тот факт, что первое слагаемое в правой части равенства (3.37) удовлетворяет этой системе точно. В результате для отыскания $\Delta $ приходим к аналогичной (3.32) задаче Коши

(3.39)
$\dot {\Delta } = A(t,\varepsilon )\Delta + B(t,\varepsilon )\Delta (t - 1) + G(t,\varepsilon ),\quad \Delta = {{\Delta }_{0}}(t,\varepsilon )\quad {\text{при}}\quad - {\kern 1pt} 1 - {{\sigma }_{0}} \leqslant t \leqslant - {{\sigma }_{0}},$
где
(3.40)
$G(t,\varepsilon ) = - \frac{{{{{\dot {\omega }}}_{2}}( - {{\sigma }_{0}},\varepsilon )}}{{{{{\dot {\omega }}}_{1}}( - {{\sigma }_{0}},\varepsilon )}}({{\Delta }_{1}}(t,\varepsilon ){{h}_{{ * * }}}(t,\varepsilon ) + {{\Delta }_{2}}(t,\varepsilon ){{h}_{{ * * }}}(t - 1,\varepsilon )),$
(3.41)
${{\Delta }_{0}}(t,\varepsilon ) = {\text{colon}}(1,0) - \frac{{\dot {\omega }(t,\varepsilon )}}{{{{{\dot {\omega }}}_{1}}( - {{\sigma }_{0}},\varepsilon )}} + \frac{{{{{\dot {\omega }}}_{2}}( - {{\sigma }_{0}},\varepsilon )}}{{{{{\dot {\omega }}}_{1}}( - {{\sigma }_{0}},\varepsilon )}}{\text{colon}}(0,1),$
а ${{\Delta }_{1}}(t,\varepsilon )$, ${{\Delta }_{2}}(t,\varepsilon )$ – матрицы из (3.29).

При анализе выражений (3.40), (3.41) обратимся к отмеченным выше асимптотическим свойствам вектор-функции $\dot {\omega }(t,\varepsilon )$, из которых следуют формулы

(3.42)
${{\dot {\omega }}_{1}}( - {{\sigma }_{0}},\varepsilon ) = 1 + O\left( {exp\left( { - \frac{q}{\varepsilon }} \right)} \right),\quad {{\dot {\omega }}_{2}}( - {{\sigma }_{0}},\varepsilon ) = 1 + O\left( {exp\left( { - \frac{q}{\varepsilon }} \right)} \right)$
и оценка
(3.43)
$\mathop {max}\limits_{ - {{\sigma }_{0}} \leqslant t \leqslant T(\varepsilon ) - {{\sigma }_{0}}} {{\left\| {\dot {\omega }(t,\varepsilon )} \right\|}_{{{{\mathbb{R}}^{2}}}}} \leqslant M,\quad M = {\text{const}} > 0.$
Опираясь, далее, на соотношения (3.30), (3.34), (3.40)–(3.43), нетрудно увидеть, что для неоднородности $G(t,\varepsilon )$ из (3.29) имеет место неравенство (3.35), а начальное условие ${{\Delta }_{0}}(t,\varepsilon )$ таково, что

(3.44)
$\mathop {max}\limits_{ - 1 - {{\sigma }_{0}} \leqslant t \leqslant - {{\sigma }_{0}}} {{\left\| {{{\Delta }_{0}}(t,\varepsilon )} \right\|}_{{{{\mathbb{R}}^{2}}}}} \leqslant M,\quad M = {\text{const}} > 0,\quad {{\Delta }_{0}}( - {{\sigma }_{0}},\varepsilon ) \equiv 0.$

Свойства (3.44) начальной вектор-функции ${{\Delta }_{0}}(t,\varepsilon )$ свидетельствуют о том, что по существу задача (3.39) отличается от рассмотренной при обосновании леммы 3.1 задачи Коши для уравнения (3.1) с начальной функцией ${{g}_{0}}(t) \in B$ лишь наличием экспоненциально малой по норме неоднородности $G(t,\varepsilon )$ (см. (3.35)). Поэтому, повторяя практически дословно соответствующий фрагмент из доказательства упомянутой леммы, убеждаемся в справедливости для решения $\Delta $ задачи Коши (3.39) оценки вида (3.36). Лемма 3.3 доказана.

Приступим теперь непосредственно к асимптотическому вычислению спектра матрицы (3.11). Опираясь на равенства (3.27), (3.38) и описанные выше приемы нахождения асимптотики вспомогательных вектор-функций ${{h}_{ * }}(t,\varepsilon )$, ${{h}_{{ * * }}}(t,\varepsilon )$, получаем серию формул

$\mathcal{D}(\varepsilon ) = \left( {\begin{array}{*{20}{c}} {{{\varkappa }_{{1,1}}}(\varepsilon )}&{{{\varkappa }_{{1,2}}}(\varepsilon )} \\ {{{\varkappa }_{{2,1}}}(\varepsilon )}&{{{\varkappa }_{{2,2}}}(\varepsilon )} \end{array}} \right),$
${{\varkappa }_{{1,1}}}(\varepsilon ) = 1 - {{\varkappa }_{{1,2}}}(\varepsilon ) + O\left( {exp\left( { - \frac{q}{\varepsilon }} \right)} \right),\quad {{\varkappa }_{{1,2}}}(\varepsilon ) = 1 - {{\left. {F{\kern 1pt} {\text{'}}(z)} \right|}_{{z = {{z}_{ * }}(a,b)}}} + O\left( {\varepsilon ln\frac{1}{\varepsilon }} \right),$
${{\varkappa }_{{2,1}}}(\varepsilon ) = O\left( {exp\left( { - \frac{q}{\varepsilon }} \right)} \right),\quad {{\varkappa }_{{2,2}}}(\varepsilon ) = 1 + O\left( {exp\left( { - \frac{q}{\varepsilon }} \right)} \right).$
Далее, принимая во внимание эту информацию и проводя соответствующий подсчет, приходим к выводу, что собственные значения $\nu _{1}^{ * }(\varepsilon )$, $\nu _{2}^{ * }(\varepsilon )$ матрицы (3.11) допускают при $\varepsilon \to 0$ асимптотику
(3.45)
$\nu _{1}^{ * }(\varepsilon ) = 1 + O\left( {exp\left( { - \frac{q}{\varepsilon }} \right)} \right),\quad \nu _{2}^{ * }(\varepsilon ) = {{\left. {F{\kern 1pt} {\text{'}}(z)} \right|}_{{z = {{z}_{ * }}(a,b)}}} + O\left( {\varepsilon ln\frac{1}{\varepsilon }} \right),$
где, напомним, ${{z}_{ * }}(a,b)$ – неподвижная точка отображения (2.42).

Для завершения доказательства теоремы 1.2 обратимся к исходному оператору (3.7) и заметим, что в силу соотношения

${{(\nu I - V(\varepsilon ))}^{{ - 1}}} = {{\left( {I - {{{(\nu I - {{V}_{2}}(\varepsilon ))}}^{{ - 1}}}{{V}_{1}}(\varepsilon )} \right)}^{{ - 1}}}{{\left( {\nu I - {{V}_{2}}(\varepsilon )} \right)}^{{ - 1}}},$
где $I$ – единичный оператор в комплексификации пространства $E$ (которую будем обозначать той же буквой $E$), любое значение $\nu \in \mathbb{C}$, для которого
(3.46)
${{\left\| {{{{\left( {\nu I - {{V}_{2}}(\varepsilon )} \right)}}^{{ - 1}}}{{V}_{1}}(\varepsilon )} \right\|}_{{E \to E}}} < 1,$
принадлежит резольвентному множеству этого оператора. Далее, напомним, что оператор ${{V}_{1}}(\varepsilon )$ допускает оценку (3.24). В случае же оператора ${{V}_{2}}(\varepsilon )$ для его резольвенты имеем явные выражения
${{(\nu I - {{V}_{2}}(\varepsilon ))}^{{ - 1}}}{{g}_{0}} = \frac{{{{g}_{0}}(t)}}{\nu } + {{r}_{1}}{{g}_{ * }}(t + T(\varepsilon ),\varepsilon ) + {{r}_{2}}{{g}_{{ * * }}}(t + T(\varepsilon ),\varepsilon ),\quad - {\kern 1pt} 1 - {{\sigma }_{0}} \leqslant t \leqslant - {{\sigma }_{0}},$
$\left( {\begin{array}{*{20}{c}} {{{r}_{1}}} \\ {{{r}_{2}}} \end{array}} \right) = {{(\nu I - \mathcal{D}(\varepsilon ))}^{{ - 1}}}\left( {\begin{array}{*{20}{c}} {{{g}_{{1,0}}}( - {{\sigma }_{0}})} \\ {{{g}_{{2,0}}}( - {{\sigma }_{0}})} \end{array}} \right),$
где $I$ – единичная $2 \times 2$ матрица, $\mathcal{D}(\varepsilon )$ – матрица (3.11). Опираясь на эти формулы и на вытекающую из (3.27), (3.38) оценку
$\mathop {max}\limits_{ - {{\sigma }_{0}} \leqslant t \leqslant T(\varepsilon ) - {{\sigma }_{0}}} \left( {{{{\left\| {{{g}_{ * }}(t,\varepsilon )} \right\|}}_{{{{\mathbb{R}}^{2}}}}} + {{{\left\| {{{g}_{{ * * }}}(t,\varepsilon )} \right\|}}_{{{{\mathbb{R}}^{2}}}}}} \right) \leqslant M,\quad M = {\text{const}} > 0,$
убеждаемся в том, что
(3.47)
${{\left\| {{{{(\nu I - {{V}_{2}}(\varepsilon ))}}^{{ - 1}}}} \right\|}_{{E \to E}}} \leqslant \frac{{M(1 + {{{\left| \nu \right|}}^{2}})}}{{\left| {\nu - \nu _{1}^{ * }(\varepsilon )} \right| \cdot \left| {\nu - \nu _{2}^{ * }(\varepsilon )} \right| \cdot \left| \nu \right|}}\quad \forall \nu \in \mathbb{C},\quad \nu \ne 0,\;\nu _{1}^{ * }(\varepsilon ),\;\nu _{2}^{ * }(\varepsilon ).$
Здесь константа $M > 0$ не зависит от $\varepsilon $, $\nu $, а через $\nu _{1}^{ * }(\varepsilon )$, $\nu _{2}^{ * }(\varepsilon )$, как и ранее, обозначены собственные значения матрицы (3.11).

Установленные оценки (3.24), (3.47) гарантируют выполнение условия (3.46) при всех $\nu \in \mathbb{C}{\backslash }\left\{ {{{O}_{1}} \cup {{O}_{2}} \cup {{O}_{3}}} \right\}$, где

(3.48)
$\begin{gathered} {{O}_{1}} = \left\{ {\nu :\left| \nu \right| < exp( - {{\delta }_{1}}{\text{/}}\varepsilon )} \right\},\quad {{O}_{2}} = \left\{ {\nu :\left| {\nu - \nu _{1}^{ * }(\varepsilon )} \right| < exp( - {{\delta }_{2}}{\text{/}}\varepsilon )} \right\}, \\ {{O}_{3}} = \left\{ {\nu :\left| {\nu - \nu _{2}^{ * }(\varepsilon )} \right| < exp( - {{\delta }_{3}}{\text{/}}\varepsilon )} \right\}, \\ \end{gathered} $
а постоянные ${{\delta }_{1}},{{\delta }_{2}},{{\delta }_{3}} > 0$ подходящим образом малы. Тем самым, спектр оператора $V(\varepsilon )$ заведомо принадлежит шарам (3.48). Более того, в силу простоты собственных значений $\nu _{1}^{ * }(\varepsilon )$, $\nu _{2}^{ * }(\varepsilon )$ матрицы (3.11) в шарах ${{O}_{2}}$ и ${{O}_{3}}$ лежит ровно по одному простому собственному значению $\nu = {{\nu }_{1}}(\varepsilon )$ и $\nu = {{\nu }_{2}}(\varepsilon )$ этого оператора, а все остальные его собственные значения ${{\nu }_{s}}(\varepsilon )$, $s \geqslant 3$ принадлежат шару ${{O}_{1}}$.

Объединяя перечисленные факты с асимптотическими представлениями (3.45), приходим к выводу, что мультипликаторы ${{\nu }_{s}}(\varepsilon )$, $s \geqslant 1$ системы (3.1) действительно обладают свойствами (3.6). Остается лишь заметить, что справедливость тождества ${{\nu }_{1}}(\varepsilon ) \equiv 1$ вытекает из факта существования у этой системы единичного мультипликатора (поскольку она представляет собой линеаризацию на цикле (2.46)). Теорема 1.2 полностью доказана.

4. МНОГОМЕРНЫЙ СЛУЧАЙ

4.1. Описание результата

Интересно отметить, что результаты, полученные нами для билокальной модели (1.6), естественным образом распространяются и на многомерную диффузионную систему

(4.1)
$\mathop {\dot {u}}\nolimits_j = D({{u}_{{j + 1}}} - 2{{u}_{{j + 1}}} + {{u}_{{j - 1}}}) + \lambda f({{u}_{j}}(t - 1)){{u}_{j}},\quad j = 1,\; \ldots ,\;m,$
где ${{u}_{0}} = {{u}_{1}}$, ${{u}_{{m + 1}}} = {{u}_{m}}$, $m \geqslant 2$.

Для удобства последующего анализа выполним в (4.1) при условии (1.8) замены переменных ${{u}_{j}} = exp(\lambda {{\omega }_{j}})$, $j = 1,\; \ldots ,\;m$, и положим $\varepsilon = 1{\text{/}}\lambda $. В результате приходим к аналогичной (2.1) системе

(4.2)
${{\dot {\omega }}_{1}} = exp\left( {\frac{{{{\omega }_{2}} - {{\omega }_{1}} - b}}{\varepsilon }} \right) - exp\left( { - \frac{b}{\varepsilon }} \right) + f\left( {exp\left( {\frac{{{{\omega }_{1}}(t - 1)}}{\varepsilon }} \right)} \right),$
(4.3)
$\begin{gathered} {{{\dot {\omega }}}_{j}} = exp\left( {\frac{{{{\omega }_{{j + 1}}} - {{\omega }_{j}} - b}}{\varepsilon }} \right) - 2exp\left( { - \frac{b}{\varepsilon }} \right) + exp\left( {\frac{{{{\omega }_{{j - 1}}} - {{\omega }_{j}} - b}}{\varepsilon }} \right) + \\ + \;f\left( {exp\left( {\frac{{{{\omega }_{j}}(t - 1)}}{\varepsilon }} \right)} \right),\quad j = 2,\; \ldots ,\;m - 1, \\ \end{gathered} $
(4.4)
${{\dot {\omega }}_{m}} = exp\left( {\frac{{{{\omega }_{{m - 1}}} - {{\omega }_{m}} - b}}{\varepsilon }} \right) - exp\left( { - \frac{b}{\varepsilon }} \right) + f\left( {exp\left( {\frac{{{{\omega }_{m}}(t - 1)}}{\varepsilon }} \right)} \right).$
Справедливо следующее утверждение.

Теорема 4.1. Пусть выполнены неравенства $a > 1$, $(m - 1)b < 1$. Тогда при всех достаточно малых $\varepsilon > 0$ и при каждом натуральном ${{j}_{0}}$, $(m + 1){\text{/}}2 \leqslant {{j}_{0}} \leqslant m$, система (4.2), (4.3) допускает экспоненциально орбитальноустойчивый цикл

(4.5)
${{\omega }_{{({{j}_{0}})}}}(t,\varepsilon ) = {\text{colon}}({{\omega }_{{({{j}_{0}}),1}}}(t,\varepsilon ),{{\omega }_{{({{j}_{0}}),2}}}(t,\varepsilon ),\; \ldots ,\;{{\omega }_{{({{j}_{0}}),m}}}(t,\varepsilon )),\quad {{\omega }_{{({{j}_{0}}),1}}}(0,\varepsilon ) \equiv 0$
периода ${{T}_{{({{j}_{0}})}}}(\varepsilon )$. При $\varepsilon \to 0$ для этого цикла справедливы асимптотические представления
(4.6)
${{T}_{{({{j}_{0}})}}}(\varepsilon ) = {{T}_{0}} + O\left( {{{\varepsilon }^{2}}ln\frac{1}{\varepsilon }} \right),\quad \mathop {max}\limits_{0 \leqslant t \leqslant {{T}_{{({{j}_{0}})}}}(\varepsilon )} \left| {{{\omega }_{{({{j}_{0}}),1}}}(t,\varepsilon ) - {{\omega }_{0}}(t)} \right| = O(\varepsilon ),$
(4.7)
$\mathop {max}\limits_{0 \leqslant t \leqslant {{T}_{{({{j}_{0}})}}}(\varepsilon )} \left| {{{\omega }_{{({{j}_{0}}),k}}}(t,\varepsilon ) - {{\omega }_{0}}(t - (k - 1){{t}_{ * }})} \right| = O\left( {\varepsilon ln\frac{1}{\varepsilon }} \right),\quad k = 2,\; \ldots ,\;{{j}_{0}},$
(4.8)
$\mathop {max}\limits_{0 \leqslant t \leqslant {{T}_{{({{j}_{0}})}}}(\varepsilon )} \left| {{{\omega }_{{({{j}_{0}}),k}}}(t,\varepsilon ) - {{\omega }_{0}}(t + (k + 1 - 2{{j}_{0}}){{t}_{ * }})} \right| = O\left( {\varepsilon ln\frac{1}{\varepsilon }} \right),\quad k = {{j}_{0}} + 1,\; \ldots ,\;m,$
где функция ${{\omega }_{0}}(t)$ и момент времени ${{t}_{ * }}$ те же самые, что и в (1.12).

Подробное доказательство теоремы 4.1 технически достаточно громоздко. Поэтому изложим лишь общую его схему. В связи с этим фиксируем произвольно индекс ${{j}_{0}}$ из промежутка $(m + 1){\text{/}}2 \leqslant {{j}_{0}} \leqslant m$ и введем в рассмотрение множество начальных функций

(4.9)
$\begin{gathered} {{S}_{{{{j}_{0}}}}} = \left\{ {\varphi (t)} \right. = {\text{colon}}({{\varphi }_{1}}(t),{{\varphi }_{2}}(t),\; \ldots ,\;{{\varphi }_{m}}(t)) \in C([ - {{\sigma }_{0}} - 1, - {{\sigma }_{0}}];{{\mathbb{R}}^{m}}): \\ - {{q}_{{1,j}}} \leqslant {{\varphi }_{j}}(t) \leqslant - {{q}_{{2,j}}},\quad j = 1,\; \ldots ,\;m,\quad {{\varphi }_{1}}( - {{\sigma }_{0}}) = - {{\sigma }_{0}},\quad \left. {{{z}_{\varphi }} \in \Omega } \right\}. \\ \end{gathered} $
Здесь, как и в (2.3), ${{q}_{{1,j}}} > {{q}_{{2,j}}} > 0$, $j = 1, \ldots ,\;m$ и ${{\sigma }_{0}} > 0$ – некоторые универсальные постоянные, $\Omega $ – выпуклый компакт из ${{\mathbb{R}}^{{m - 1}}}$, а функционал ${{z}_{\varphi }}$ задается равенствами
(4.10)
${{z}_{\varphi }} = {\text{colon}}({{z}_{{1,\varphi }}},{{z}_{{2,\varphi }}},\; \ldots ,\;{{z}_{{m - 1,\varphi }}}),$
(4.11)
$\begin{gathered} {{z}_{{k,\varphi }}} = \frac{1}{\varepsilon }\left( {{{\varphi }_{{k + 1}}}( - {{\sigma }_{0}}) + {{\sigma }_{0}} + \left( {\frac{b}{a} - \frac{\varepsilon }{a}ln\frac{1}{\varepsilon }} \right)k} \right),\quad k = 1,\; \ldots ,\;{{j}_{0}} - 1, \\ {{z}_{{k,\varphi }}} = \frac{1}{\varepsilon }\left( {{{\varphi }_{{k + 1}}}( - {{\sigma }_{0}}) + {{\sigma }_{0}} + \left( {\frac{b}{a} - \frac{\varepsilon }{a}ln\frac{1}{\varepsilon }} \right)(2{{j}_{0}} - k - 2)} \right),\quad k = {{j}_{0}},\; \ldots ,\;m - 1. \\ \end{gathered} $
Далее, рассмотрим решение ${{\omega }_{\varphi }}(t,\varepsilon ) = {\text{colon}}({{\omega }_{{1,\varphi }}}(t,\varepsilon ),{{\omega }_{{2,\varphi }}}(t,\varepsilon ),\; \ldots ,\;{{\omega }_{{m,\varphi }}}(t,\varepsilon ))$, $t \geqslant - {{\sigma }_{0}}$, системы (4.2)–(4.4) с произвольным начальным условием $\varphi (t) \in {{S}_{{{{j}_{0}}}}}$ при $ - 1 - {{\sigma }_{0}} \leqslant t \leqslant - {{\sigma }_{0}}$, обозначим через $t = {{T}_{\varphi }}(\varepsilon )$ второй положительный корень уравнения (2.6) и зададим оператор $\Pi $ с помощью формулы (2.7).

Асимптотический анализ решения ${{\omega }_{\varphi }}(t,\varepsilon )$ на отрезке (2.8) базируется на априорных оценках

(4.12)
$\begin{gathered} {{\omega }_{{k - 1,\varphi }}}(t,\varepsilon ) - {{\omega }_{{k,\varphi }}}(t,\varepsilon ) - b \leqslant - {{M}_{1}},\quad k = 1,2,\; \ldots ,\;{{j}_{0}}, \\ {{\omega }_{{k + 1,\varphi }}}(t,\varepsilon ) - {{\omega }_{{k,\varphi }}}(t,\varepsilon ) - b \leqslant - {{M}_{2}},\quad k = {{j}_{0}},\; \ldots ,\;m - 1, \\ \end{gathered} $
где ${{M}_{1}},{{M}_{2}} = {\text{const}} > 0$. Учитывая неравенства (4.12) в правых частях системы (4.2)–(4.4), приходим к выводу, что она приобретает вид

(4.13)
${{\dot {\omega }}_{k}} = exp\left( {\frac{{{{\omega }_{{k + 1}}} - {{\omega }_{k}} - b}}{\varepsilon }} \right) + f\left( {exp\left( {\frac{{{{\omega }_{k}}(t - 1)}}{\varepsilon }} \right)} \right) + O\left( {exp\left( { - \frac{q}{\varepsilon }} \right)} \right),\quad k = 1,2,\; \ldots ,\;{{j}_{0}} - 1,$
(4.14)
${{\dot {\omega }}_{{{{j}_{0}}}}} = f\left( {exp\left( {\frac{{{{\omega }_{{{{j}_{0}}}}}(t - 1)}}{\varepsilon }} \right)} \right) + O\left( {exp\left( { - \frac{q}{\varepsilon }} \right)} \right),$
(4.15)
${{\dot {\omega }}_{k}} = exp\left( {\frac{{{{\omega }_{{k - 1}}} - {{\omega }_{k}} - b}}{\varepsilon }} \right) + f\left( {exp\left( {\frac{{{{\omega }_{k}}(t - 1)}}{\varepsilon }} \right)} \right) + O\left( {exp\left( { - \frac{q}{\varepsilon }} \right)} \right),\quad k = {{j}_{0}} + 1,\; \ldots ,\;m.$

Последующие построения аналогичны изложенным в п. 2.2. Действительно, так как уравнение (4.14) с экспоненциальной точностью отщепляется от всей системы (4.13)–(4.15), то сначала находим асимптотику на отрезке (2.8) компоненты ${{\omega }_{{{{j}_{0}},\varphi }}}(t,\varepsilon )$. Нетрудно увидеть, что для нее получается серия формул, аналогичных (2.13)–(2.22) (с заменой момента времени (2.12) на ${{t}_{{ * ,{{j}_{0}}}}}(\varepsilon ) = ({{t}_{ * }} - (\varepsilon {\text{/}}a)ln(1{\text{/}}\varepsilon ))({{j}_{0}} - 1) - \varepsilon {{z}_{{{{j}_{0}} - 1}}}$, где ${{z}_{{{{j}_{0}} - 1}}}$ – соответствующая компонента из (4.11)). Далее, зная асимптотику ${{\omega }_{{{{j}_{0}},\varphi }}}(t,\varepsilon )$ и обращаясь к уравнениям из (4.13) и (4.15) при $k = {{j}_{0}} - 1$ и $k = {{j}_{0}} + 1$, вычисляем асимптотику компонент ${{\omega }_{{{{j}_{0}} - 1,\varphi }}}(t,\varepsilon )$ и ${{\omega }_{{{{j}_{0}} + 1,\varphi }}}(t,\varepsilon )$. Затем, принимая во внимание эту информацию, из уравнений (4.13), (4.15) с номерами $k = {{j}_{0}} - 2$ и $k = {{j}_{0}} + 2$ находим асимптотику ${{\omega }_{{{{j}_{0}} - 2,\varphi }}}(t,\varepsilon )$, ${{\omega }_{{{{j}_{0}} + 2,\varphi }}}(t,\varepsilon )$ и т.д. На завершающем этапе, используя полученные в результате описанных действий асимптотические формулы, убеждаемся в том, что при подходящем выборе констант ${{M}_{1}}$, ${{M}_{2}} > 0$ оценки (4.12) действительно выполняются.

Опираясь на изложенную схему асимптотического анализа решения ${{\omega }_{\varphi }}(t,\varepsilon )$, удается показать, что вопросы о существовании и устойчивости цикла (4.5) с асимптотикой (4.6)–(4.8) сводятся к отысканию неподвижных точек $(m - 1)$-мерного отображения, аналогичного (2.42):

(4.16)
$z \mapsto F(z) = {\text{colon}}({{F}_{1}}(z),{{F}_{2}}(z),\; \ldots ,\;{{F}_{{m - 1}}}(z)).$
Здесь $z = {\text{colon}}({{z}_{1}},{{z}_{2}},\; \ldots ,\;{{z}_{{m - 1}}})$ – функционал (4.10) с компонентами (4.11) (индекс $\varphi $ для краткости мы опускаем), а функции ${{F}_{j}}(z)$, $j = 1,\; \ldots ,\;m - 1$, имеют вид

(4.17)
${{F}_{j}}(z) = {{\Delta }_{j}}(z) - {{\Delta }_{0}}(z),\quad j = 1,\; \ldots ,\;m - 1,$
(4.18)
$\begin{gathered} {{\Delta }_{s}}(z) = ln\left( {\sum\limits_{k = s}^{{{j}_{0}} - 1} {exp( - a{{z}_{k}})} \frac{{{{{({{t}_{0}} - (k - s){{t}_{ * }})}}^{{k - s}}}}}{{(k - s)!}}} \right) - \left( {1 + \frac{1}{a}} \right)ln\left( {\sum\limits_{k = s}^{{{j}_{0}} - 1} {exp( - a{{z}_{k}})} \frac{{{{{({{t}_{0}} - 1 - (k - s){{t}_{ * }})}}^{{k - s}}}}}{{(k - s)!}}} \right), \\ s = 0,1,\; \ldots ,\;{{j}_{0}} - 1, \\ \end{gathered} $
(4.19)
$\begin{gathered} {{\Delta }_{{m - s}}}(z) = ln\left( {\sum\limits_{k = s}^{m - {{j}_{0}} + 1} {exp( - a{{z}_{{m - k}}})} \frac{{{{{({{t}_{0}} - (k - s){{t}_{ * }})}}^{{k - s}}}}}{{(k - s)!}}} \right) - \\ - \;\left( {1 + \frac{1}{a}} \right)ln\left( {\sum\limits_{k = s}^{m - {{j}_{0}} + 1} {exp( - a{{z}_{{m - k}}})} \frac{{{{{({{t}_{0}} - 1 - (k - s){{t}_{ * }})}}^{{k - s}}}}}{{(k - s)!}}} \right),\quad s = 1,\; \ldots ,\;m - {{j}_{0}}. \\ \end{gathered} $

Анализ отображения (4.16) существенно упрощается после выполнения в нем замен переменных

(4.20)
${{x}_{{{{j}_{0}} - k}}} = {{z}_{{{{j}_{0}} - k - 1}}} - {{z}_{{{{j}_{0}} - 1}}},\quad k = 1,\; \ldots ,\;{{j}_{0}} - 1;\quad {{y}_{k}} = {{z}_{{{{j}_{0}} + k}}} - {{z}_{{{{j}_{0}} - 1}}},\quad k = 0,1,\; \ldots ,\;m - 1 - {{j}_{0}},$
где ${{x}_{{{{j}_{0}}}}} = 0$, ${{y}_{{ - 1}}} = 0$, ${{z}_{0}} = 0$. Действительно, переходя к новым координатам (4.20) и принимая во внимание формулы (4.17)(4.19), приходим к выводу, что исходное отображение (4.16) распадается на два независимых треугольных отображения

(4.21)
$\begin{gathered} {{x}_{{{{j}_{0}} - k}}} \mapsto ln\left( {\sum\limits_{r = {{j}_{0}} - k - 1}^{{{j}_{0}} - 1} {exp( - a{{x}_{{r + 1}}})} \frac{{{{{({{t}_{0}} - (r - {{j}_{0}} + k + 1){{t}_{ * }})}}^{{r - {{j}_{0}} + k + 1}}}}}{{(r - {{j}_{0}} + k + 1)!}}} \right) - \\ - \;\left( {1 + \frac{1}{a}} \right)ln\left( {\sum\limits_{r = {{j}_{0}} - k - 1}^{{{j}_{0}} - 1} {exp( - a{{x}_{{r + 1}}})} \frac{{{{{({{t}_{0}} - 1 - (r - {{j}_{0}} + k + 1){{t}_{ * }})}}^{{r - {{j}_{0}} + k + 1}}}}}{{(r - {{j}_{0}} + k + 1)!}}} \right), \\ k = 1,\; \ldots ,\;{{j}_{0}} - 1; \\ \end{gathered} $
(4.22)
$\begin{gathered} {{y}_{k}} \mapsto ln\left( {\sum\limits_{r = 0}^{k + 1} {exp( - a{{y}_{{k - r}}})} \frac{{{{{({{t}_{0}} - r{{t}_{ * }})}}^{r}}}}{{r!}}} \right) - \\ - \;\left( {1 + \frac{1}{a}} \right)ln\left( {\sum\limits_{r = 0}^{k + 1} {exp( - a{{y}_{{k - r}}})} \frac{{{{{({{t}_{0}} - 1 - r{{t}_{ * }})}}^{r}}}}{{r!}}} \right),\quad k = 0,1,\; \ldots ,\;m - 1 - {{j}_{0}}. \\ \end{gathered} $

Обратимся сначала к отображению (4.21) и заметим, что от него отщепляется одномерное отображение

(4.23)
${{x}_{{{{j}_{0}} - 1}}} \mapsto ln(r_{{1,1}}^{ * } + exp( - a{{x}_{{{{j}_{0}} - 1}}})) - \left( {1 + \frac{1}{a}} \right)ln(r_{{2,1}}^{ * } + exp( - a{{x}_{{{{j}_{0}} - 1}}})),$
где $r_{{1,1}}^{ * } = {{t}_{0}} - {{t}_{ * }}$, $r_{{2,1}}^{ * } = {{t}_{0}} - {{t}_{ * }} - 1$. Как уже отмечалось выше, это отображение является частным случаем (2.43) и в силу неравенства (2.44) (имеющего место при ${{r}_{1}} = r_{{1,1}}^{ * }$, ${{r}_{2}} = r_{{2,1}}^{ * }$) допускает единственную экспоненциально устойчивую неподвижную точку ${{x}_{{{{j}_{0}} - 1}}} = x_{{{{j}_{0}} - 1}}^{ * }$.

Предположим теперь, что сделано $k - 1$ шагов и найдены компоненты ${{x}_{{{{j}_{0}} - s}}} = x_{{{{j}_{0}} - s}}^{ * }$, $s = 1,\; \ldots ,\;k - 1$, устойчивой неподвижной точки отображения (4.21). Тогда, подставляя эти компоненты в (4.21), для определения ${{x}_{{{{j}_{0}} - k}}}$ получаем аналогичное (4.23) одномерное отображение

(4.24)
${{x}_{{{{j}_{0}} - k}}} \mapsto ln(r_{{1,k}}^{ * } + exp( - a{{x}_{{{{j}_{0}} - k}}})) - \left( {1 + \frac{1}{a}} \right)ln(r_{{2,k}}^{ * } + exp( - a{{x}_{{{{j}_{0}} - k}}})),$
где
$r_{{1,k}}^{ * } = \sum\limits_{r = {{j}_{0}} - k}^{{{j}_{0}} - 1} {exp( - ax_{{r + 1}}^{ * })} \frac{{{{{({{t}_{0}} - (r - {{j}_{0}} + k + 1){{t}_{ * }})}}^{{r - {{j}_{0}} + k + 1}}}}}{{(r - {{j}_{0}} + k + 1)!}},$
$r_{{2,k}}^{ * } = \sum\limits_{r = {{j}_{0}} - k}^{{{j}_{0}} - 1} exp( - ax_{{r + 1}}^{ * })\frac{{{{{({{t}_{0}} - 1 - (r - {{j}_{0}} + k + 1){{t}_{ * }})}}^{{r - {{j}_{0}} + k + 1}}}}}{{(r - {{j}_{0}} + k + 1)!}}.$
Заметим, далее, что и здесь требование вида (2.44) остается в силе, поскольку
$\begin{gathered} r_{{1,k}}^{ * } - \left( {1 + \frac{1}{a}} \right)r_{{2,k}}^{ * } = \sum\limits_{r = {{j}_{0}} - k}^{{{j}_{0}} - 1} {exp( - ax_{{r + 1}}^{ * })} \left\{ {\frac{{{{{({{t}_{0}} - (r - {{j}_{0}} + k + 1){{t}_{ * }})}}^{{r - {{j}_{0}} + k + 1}}}}}{{(r - {{j}_{0}} + k + 1)!}} - } \right. \\ \left. { - \;\left( {1 + \frac{1}{a}} \right)\frac{{{{{({{t}_{0}} - 1 - (r - {{j}_{0}} + k + 1){{t}_{ * }})}}^{{r - {{j}_{0}} + k + 1}}}}}{{(r - {{j}_{0}} + k + 1)!}}} \right\} \\ \end{gathered} $
и справедливы вытекающие из условий $a > 1$, $(m - 1)b < 1$ оценки
$\begin{gathered} {{t}_{0}} - 1 - (r - {{j}_{0}} + k + 1){{t}_{ * }} > 0,\quad r = {{j}_{0}} - k,\; \ldots ,\;{{j}_{0}} - 1, \\ {{({{t}_{0}} - (r - {{j}_{0}} + k + 1){{t}_{ * }})}^{{r - {{j}_{0}} + k + 1}}} > \left( {1 + \frac{1}{a}} \right){{({{t}_{0}} - 1 - (r - {{j}_{0}} + k + 1){{t}_{ * }})}^{{r - {{j}_{0}} + k + 1}}}. \\ \end{gathered} $
А это значит, что отображение (4.24) имеет единственную экспоненциально устойчивую неподвижную точку ${{x}_{{{{j}_{0}} - k}}} = x_{{{{j}_{0}} - k}}^{ * }$.

Продолжая описанный процесс вплоть до номера $k = {{j}_{0}} - 1$, убеждаемся в том, что исходное отображение (4.21) допускает единственную экспоненциально устойчивую неподвижную точку $(x_{1}^{ * },\; \ldots ,\;x_{{{{j}_{0}} - 1}}^{ * })$. Добавим еще, что отображение (4.22) анализируется аналогично и также имеет единственную устойчивую неподвижную точку $(y_{0}^{ * },y_{1}^{ * }, \ldots ,\;y_{{m - 1 - {{j}_{0}}}}^{ * })$.

Подводя итог, отметим, что установленная информация об отображении (4.16) позволяет завершить доказательство теоремы 4.1. Действительно, опираясь на факт существования у этого отображения устойчивой неподвижной точки, нетрудно показать, что при подходящем выборе постоянных ${{\sigma }_{0}}$, ${{q}_{{1,j}}}$, ${{q}_{{1,j}}}$, $j = 1,\; \ldots ,\;m$, и компакта $\Omega \subset {{\mathbb{R}}^{{m - 1}}}$ оператор $\Pi $ переводит множество (4.9) в себя и, следовательно,  допускает в нем неподвижную точку. Что же касается цикла системы (4.2)–(4.4), отвечающего данной неподвижной точке, то он обладает требуемой асимптотикой (4.6)–(4.8) и является устойчивым (доказательство этих фактов проводится по той же схеме, что и при $m = 2$).

4.2. Заключение

Отдельного  упоминания  заслуживает следующее обстоятельство. Непосредственная проверка показывает, что система (4.2)–(4.4) инвариантна по отношению к замене переменных ${{x}_{{m + 1 - j}}} \to {{x}_{j}}$, $j = 1,\; \ldots ,\;m$. Поэтому наряду с периодическими решениями (4.5) при $(m + 1){\text{/}}2 \leqslant {{j}_{0}} \leqslant m$ у нее существуют также циклы, получающиеся из данных циклов при указанном преобразовании координат. Общее же количество устойчивых автоволновых периодических режимов, существующих у системы (4.2)–(4.4) в рамках условий теоремы 4.1, равно $m$ (в случае $m = 2{{m}_{0}} + 1$ цикл (4.5) с номером ${{j}_{0}} = {{m}_{0}} + 1$ при упомянутой выше замене переменных переходит сам в себя).

В заключение остановимся на биологической интерпретации периодического режима (4.5). Из формул (4.6)–(4.8) и равенств ${{u}_{j}} = exp(\lambda {{\omega }_{j}})$, $j = 1,\; \ldots ,\;m$, вытекает, что поведение компонент ${{u}_{j}}$, $j = 1, \ldots ,\;m$, отвечающих данному режиму, характеризуется следующими особенностями.

При увеличении переменной $t$ и при прохождении ее через значение $t = 0$ сначала активируется первый нейрон (начинается всплеск компоненты ${{u}_{1}}$), а остальные нейроны не возбуждены (компоненты ${{u}_{j}}$, $j \geqslant 2$ экспоненциально малы, т.е. имеют порядок $exp( - q\lambda )$, $q = {\text{const}} > 0$). При дальнейшем увеличении $t$ и при достижении момента $t = {{t}_{ * }}$ начинается всплеск компоненты ${{u}_{2}}$ (компонента ${{u}_{1}}$ при этом экспоненциально велика, т.е. имеет порядок $exp(q\lambda )$, $q = {\text{const}} > 0$). При $t \approx 2{{t}_{ * }}$ активируется третий нейрон, т.е. становится экспоненциально большой компонента ${{u}_{3}}$, и т.д. (вплоть до момента $t = ({{j}_{0}} - 1){{t}_{ * }}$). Следует также отметить, что при прохождении $t$ через значения $t = (j_{0}^{'} + k){{t}_{ * }}$, $k = 0,1,\; \ldots ,\;m - {{j}_{0}}$, где $j_{0}^{'} = 2{{j}_{0}} - m - 1$, активируется сразу пара нейронов с номерами $j = j_{0}^{'} + k + 1$ и $j = m - k$ соответственно (за исключением случая $k = m - {{j}_{0}}$, $t \approx ({{j}_{0}} - 1){{t}_{ * }}$, когда становится экспоненциально большой только одна компонента ${{u}_{{{{j}_{0}}}}}$).

При последующих $t$, а именно, при $({{j}_{0}} - 1){{t}_{ * }} \lesssim t \lesssim {{t}_{0}}$ активна вся нейронная система, а затем начинается процесс падения компонент ${{u}_{j}}$ от экспоненциально больших до экспоненциально малых значений. Указанное падение осуществляется при прохождении $t$ через моменты времени $t = k{{t}_{ * }} + {{t}_{0}}$, $k = 0,1,\; \ldots ,\;{{j}_{0}} - 1$, и происходит в том же порядке индексов, что и активация. В итоге на промежутке времени $({{j}_{0}} - 1){{t}_{ * }} + {{t}_{0}} \lesssim t \lesssim {{T}_{0}}$ становятся экспоненциально малыми все компоненты ${{u}_{j}}$, $j = 1,\; \ldots ,\;m$. При $t \gtrsim {{T}_{0}}$ весь процесс активации повторяется.

Обратим внимание на особую роль нейрона с номером $j = {{j}_{0}}$, который служит неким поглощающим центром. Действительно, волны возбуждения, начинающиеся с концов данной нейронной цепочки, движутся навстречу друг другу к точке $j = {{j}_{0}}$ и гасятся в этой точке.

Список литературы

  1. Глызин С.Д., Колесов А.Ю., Розов Н.Х. Об одном способе математического моделирования химических синапсов // Дифференц. ур-ния. 2013. Т. 49. № 10. С. 1227–1244.

  2. Глызин С.Д., Колесов А.Ю., Розов Н.Х. Релаксационные автоколебания в сетях импульсных нейронов // Успехи матем. наук. 2015. Т. 70. Вып. 3(423). С. 3–76.

  3. Hutchinson G.E. Circular causal systems in ecology // Ann. N. Y. Acad. of Sci. 1948. V. 50. P. 221–246.

  4. Колесов А.Ю., Мищенко Е.Ф., Розов Н.Х. Об одной модификации уравнения Хатчинсона // Ж. вычисл. матем. и матем. физ. 2010. Т. 50. № 12. С. 2099–2112.

  5. Antoniou I., Prigogine I., Sadovnichii V., Shkarin S.A. Time operator for diffusion // Chaos, Solitons and Fractals. 2000. V. 11. P. 465–477.

  6. Колесов А.Ю., Розов Н.Х. Автоволновые процессы в цепочках диффузионно связанных уравнений с запаздыванием // Успехи матем. наук. 2012. Т. 67. Вып 2(404). С. 109–156.

  7. Глызин С.Д., Колесов А.Ю., Розов Н.Х. Устойчивый релаксационный цикл в билокальной нейронной модели // Дифференц. ур-ния. 2018. Т. 54. № 10. С. 1313–1337.

Дополнительные материалы отсутствуют.